Archivo del Autor: Pedro Rivera Herrera

Variable Compleja I: Integrales de contorno I

Por Pedro Rivera Herrera

Introducción

En la entrada anterior vimos la definición de la integral para funciones complejas de variable real, es decir, funciones híbridas. Aunque de cierta manera esta definición nos limita, ya que en general trabajamos con funciones complejas de variable compleja.

Al igual que sucedió con el concepto de diferenciabilidad para una función compleja de variable compleja, también existe el concepto de integrabilidad para funciones complejas. En esta entrada veremos que aunque muchas de las definiciones y resultados para este tipo de integrales son una extensión de los conceptos de integración para funciones de varias variables reales, vistos en nuestros cursos de Cálculo, la integración en el sentido complejo va más allá de un simple salto de los resultados para funciones reales a la variable compleja, ya que como veremos, a través de la integración compleja es posible obtener herramientas e ideas únicas para el estudio de la teoría de las funciones complejas.

Definición 34.1. (Integral de contorno o integral de línea compleja.)
Sean $U\subset\mathbb{C}$ un conjunto abierto, $[a,b]\subset\mathbb{R}$, con $a<b$ un intervalo cerrado, $f: U \to \mathbb{C}$ una función continua en $U$ y $\gamma:[a,b] \to U$ un contorno en $U$ (definición 32.9). Se define a la integral de contorno o integral de línea compleja, a lo largo de $\gamma$, como:
\begin{equation*}
\int_{\gamma} f(z) dz := \int_{a}^{b} f(\gamma(t)) \gamma'(t)dt.\tag{34.1}
\end{equation*}

Si $C$ denota al contorno dado por la trayectoria $\gamma$, entonces la integral en (34.1) se puede escribir como:
\begin{equation*}
\int_{C} f(z) dz.
\end{equation*}

Observación 34.1.
Recordemos que un contorno es una trayectoria $\gamma$ de clase $C^1$ o de clase $C^1$ a trozos, por lo que al igual que con las integrales de funciones híbridas, esta definición ya considera el caso en el que $\gamma$ sea una curva suave a trozos. En tal caso, para la partición:
\begin{equation*}
P : a=t_0 < t_1 < \cdots < t_{n-1}<t_n=b,
\end{equation*}del intervalo $[a,b]$, tal que $\gamma_k$, la restricción de $\gamma$ al intervalo $[t_{k-1}, t_k]$, es una curva suave para $1\leq k\leq n$, entonces:
\begin{equation*}
\int_{\gamma} f(z) dz = \int_{\gamma_1} f(z) dz + \cdots + \int_{\gamma_n} f(z) dz = \sum_{k=1}^n \int_{\gamma_k} f(z) dz. \tag{34.2}
\end{equation*}

Observación 34.2.
Si $f(z) = u(z)+iv(z)$ y $\gamma(t)=\gamma_1(t)+i\gamma_2(t)$, tenemos que:
\begin{align*}
f(\gamma(t)) \gamma'(t) & = \left[u(\gamma(t))+iv(\gamma(t))\right]\left[\gamma_1 ‘(t)+i\gamma_2′(t)\right]\\
& = u(\gamma(t)) \gamma_1 ‘(t) – v(\gamma(t)) \gamma_2′(t) + i \left[u(\gamma(t)) \gamma_2 ‘(t) + v(\gamma(t)) \gamma_1′(t)\right],
\end{align*}por lo que la función híbrida $g(t)= f(\gamma(t)) \gamma'(t)$ es continua (o continua a trozos) en $[a,b]$, entonces la integral del lado derecho en (34.1) está bien definida.

Ejemplo 34.1.
Sea $C$ el contorno dado por la circunferencia $C(z_0, r)$, con $r>0$ y $z_0\in\mathbb{C}$ fijo, orientada positivamente.

a) Veamos que:
\begin{equation*}
\int_{C} \frac{1}{z-z_0} dz = i 2\pi.
\end{equation*}

b) Si $n\in\mathbb{Z}$ es tal que $n\neq1$, veamos que:
\begin{equation*}
\int_{C} \frac{1}{(z-z_0)^n} dz = 0.
\end{equation*}

Solución. Primeramente, podemos parametrizar al contorno $C$ mediante la trayectoria $\gamma(t)=z_0 + re^{it}$, con $0\leq t\leq 2\pi$. Por la proposición 32.1(1) y el ejemplo 32.1 tenemos que $\gamma'(t)=ire^{it}$.

Sea $D := \mathbb{C}\setminus\{z_0\}$. Claramente $C$ es un contorno en $D$.

Figura 122: Contorno $C$ dado por la circunferencia $C(z_0, r)$, orientada positivamente, en el dominio $D$.

a) Sea $f(z)=\dfrac{1}{z-z_0}$. Dado que $f$ es una función racional, entonces es analítica en el dominio $D$ y por tanto continua en $D$.

De acuerdo con la definición 34.1, tenemos que:
\begin{align*}
\int_{C} \frac{1}{z-z_0} dz & = \int_{0}^{2\pi} f(\gamma(t)) \gamma'(t)dt\\
& = \int_{0}^{2\pi} \frac{1}{z_0 +re^{it} – z_0} ire^{it} dt\\
& = \int_{0}^{2\pi} i dt\\
& = i 2\pi.
\end{align*}

b) Sean $n\in\mathbb{Z}$ tal que $n\neq1$ y $f(z)=\dfrac{1}{(z-z_0)^n}$. Análogamente tenemos que la función racional $f$ es continua en $D$.

Considerando la definición 34.1, el ejemplo 32.1 y las proposiciones 33.1(3), 33.2, 20.2(2) y 20.2(10), tenemos que:
\begin{align*}
\int_{C} \frac{1}{(z-z_0)^n} dz & = \int_{0}^{2\pi} f(\gamma(t)) \gamma'(t)dt\\
%& = \int_{0}^{2\pi} \frac{1}{(z_0 +re^{it} – z_0)^n} ire^{it} dt\\
& = ir^{1-n} \int_{0}^{2\pi} e^{it(1-n)} dt\\
& = ir^{1-n} \left. \frac{e^{it(1-n)}}{i(1-n)} \right|_{0}^{2\pi}\\
& = \frac{r^{1-n}}{1-n} \left(e^{i2\pi(1-n)} – e^{0}\right)\\
& = \frac{r^{1-n}}{1-n} \left(1 – 1\right)\\
& = 0.
\end{align*}

En particular, si $C$ es la circunferencia unitaria, orientada positivamente, es decir, dada por la trayectoria $\gamma(t)=e^{it}$, con $0\leq t\leq 2\pi$, entonces se cumple que:
\begin{equation*}
\int_{C} \frac{1}{z} dz = \int_{C(0,1)} \frac{1}{z} dz = 2\pi i.
\end{equation*}

Ejemplo 34.2.
De acuerdo con los ejemplos 32.1, 33.2 y las proposiciones 20.2(2) y 33.1(3), para el contorno $C$ dado por la circunferencia unitaria, orientada positivamente, es decir, $\gamma(t)=e^{it}$, con $0\leq t\leq 2\pi$, tenemos que:
\begin{align*}
\int_{C(0,1)} z^{n} dz & = \int_{0}^{2\pi} e^{itn} i e^{it} dt\\
& = i \int_{0}^{2\pi} e^{it(n+1)} dt\\
& = \left\{ \begin{array}{lcc}
0 & \text{si} & n \neq -1, \\ \\
i2\pi & \text{si} & n=-1.
\end{array} \right.
\end{align*}para todo $n\in\mathbb{Z}$.

Ejemplo 34.3.
Sea $C$ el contorno dado por la circunferencia $C(0, 1)$, orientada positivamente. Veamos que:
\begin{equation*}
\int_{C} \frac{1}{\overline{z}} dz = 0 \quad \text{y} \quad \int_{C} \overline{z} dz = i2\pi.
\end{equation*}

Solución. Podemos parametrizar a $C$ como la trayectoria $\gamma(t)=e^{it}$, con $0\leq t\leq 2\pi$, por lo que $\gamma'(t)=ie^{it}$.

Por la proposición 20.2(8) tenemos que $\overline{e^{it}} = e^{\overline{it}} = e^{-it}$, entonces, de acuerdo con la definición 34.1, el ejemplo 33.2 y las proposiciones 20.2(2), 20.2(3) y 33.1(3), tenemos que:
\begin{align*}
\int_{C(0,1)} \frac{1}{\overline{z}} dz & = \int_{0}^{2\pi} \frac{1}{\overline{e^{it}}} i e^{it} dt\\
& = \int_{0}^{2\pi} \frac{1}{e^{-it}} i e^{it} dt\\
& = i \int_{0}^{2\pi} e^{i2t} dt\\
& = i \cdot 0\\
& = 0.
\end{align*}
\begin{align*}
\int_{C(0,1)}\overline{z} dz & = \int_{0}^{2\pi} \overline{e^{it}} i e^{it} dt\\
& = \int_{0}^{2\pi} e^{-it} i e^{it} dt\\
& = i \int_{0}^{2\pi} 1 dt\\
& = i 2\pi.
\end{align*}

Ejemplo 34.4.
Evaluemos la integral $\int_{\gamma} (x+y) dz$ a lo largo del contorno $\gamma=[0,1+i]+[1+i, i]$.

Solución. De acuerdo con el ejemplo 15.1, es claro que para $z=x+iy\in\mathbb{C}$ la función $f(z)=\operatorname{Re}(z)+\operatorname{Im}(z)$ es una función continua en $\mathbb{C}$. Notemos que el contorno dado por la trayectoria $\gamma$ es una curva suave a trozos. Por el ejemplo 32.2 tenemos que:
\begin{equation*}
[0,1+i](t) = (1+i)t, \quad [1+i,i](t) = 1+i -t, \quad \forall t\in[0,1].
\end{equation*}

De acuerdo con la definición 32.13, tenemos que:
\begin{equation*}
\gamma(t) : = \left( [0,1+i] + [1+i,i] \right)(t) = \left\{ \begin{array}{lcc} (1+i)t & \text{si} & 0 \leq t \leq 1, \\ \\
2-t+i & \text{si} & 1 \leq t \leq 2. \end{array} \right.
\end{equation*}

Es claro que las curvas $\gamma_1 = \left. \gamma\right|_{[0,1]}$ y $\gamma_2 = \left. \gamma\right|_{[1,2]}$ son suaves, cuyas derivadas son, respectivamente, $\gamma_1′(t) = 1+i$ y $\gamma_2′(t) = -1$.

Entonces, por (34.2), (34.1) y las proposiciones 33.1 y 33.2, se tiene que:
\begin{align*}
\int_{\gamma} (x+y) dz & = \int_{\gamma_1} (x+y) dz + \int_{\gamma_2} (x+y) dz\\
& = \int_{0}^{1} 2t(1+i) dt + \int_{1}^{2} (3-t)(-1) dt\\
&= \left.(1+i)t^2\right|_{0}^{1} – \left.\left[3t – \frac{t^2}{2}\right]\right|_{1}^{2}\\
& = -\frac{1}{2} + i.
\end{align*}

Observación 34.3.
Considerando la definición 33.1 y el producto interior de $\mathbb{R}^2$, tenemos que:
\begin{align*}
\int_{\gamma} f(z) dz & = \int_{a}^{b} f(\gamma(t)) \gamma'(t) dt\\
& = \int_{a}^{b} \left[u(\gamma(t)) \gamma_1 ‘(t) – v(\gamma(t)) \gamma_2′(t)\right] dt + i \int_{a}^{b} \left[v(\gamma(t)) \gamma_1′(t) + u(\gamma(t)) \gamma_2 ‘(t)\right] dt\\
& = \int_{a}^{b} \left(u(\gamma(t)), -v(\gamma(t))\right) \cdot \left(\gamma_1 ‘(t), \gamma_2′(t)\right) dt + i \int_{a}^{b} \left(v(\gamma(t)), u(\gamma(t))\right) \cdot \left( \gamma_1′(t), \gamma_2 ‘(t)\right) dt.
\end{align*}

Si definimos a los campos vectoriales, en el plano, $F, G: U\subset\mathbb{R}^2 \to \mathbb{R}^2$ dados, respectivamente, por:
\begin{equation*}
F(x,y)=(u(x,y), -v(x,y)) \quad \text{y} \quad G(x,y)=(v(x,y), u(x,y)),
\end{equation*}entonces:
\begin{align*}
\int_{\gamma} f(z) dz & = \int_{a}^{b} F(\gamma(t)) \cdot \gamma'(t) dt + i \int_{a}^{b} G(\gamma(t)) \cdot \gamma'(t) dt\\
& = \int_\Gamma F \cdot d\gamma + i \int_\Gamma G \cdot d\gamma,
\end{align*}donde $\Gamma=\gamma([a,b])$. Es decir, la interal que definimos en 34.1 se puede expresar en términos de la integral de línea de dos campos vectoriales en $\mathbb{R}^2$.

En este punto es conveniente recordar el siguiente resultado de Cálculo.

Teorema 34.1. (Cambio de variable.)
Sean $[a,b], [c,d]\subset\mathbb{R}$, con $a<b$ y $c<d$, dos intervalos cerrados, $f: [a,b] \to \mathbb{R}$ y $g: [c,d] \to \mathbb{R}$ dos funciones tales que $g([c,d])\subseteq [a,b]$, $f$ es continua en $[a,b]$ y $g$ de clase $C^1$ en $[c,d]$, entonces:
\begin{equation*}
\int_{c}^{d} f(g(t)) g'(t) dt = \int_{g(c)}^{g(d)} f(t)dt.
\end{equation*}

Una consecuencia del resultado anterior es la siguiente:

Proposición 34.1. (Independencia de la parametrización.)
Sean $U\subset\mathbb{C}$ un conjunto abierto, $[a,b], [c,d]\subset\mathbb{R}$, con $a<b$ y $c<d$, dos intervalos cerrados, $f: U \to \mathbb{C}$ una función continua en $U$ y $\gamma_1:[a,b] \to U$ un contorno en $U$. Si $\gamma_2:[c,d] \to U$ es una reparametrización de $\gamma_1$, entonces:
\begin{equation*}
\int_{\gamma_1} f(z) dz = \int_{\gamma_2} f(z) dz.
\end{equation*}

Demostración. Dadas las hipótesis, tenemos que existe una biyección $\sigma:[c,d]\to[a,b]$ continua de clase $C^1$ tal que $\sigma$ es creciente y $\gamma_2=\gamma_1\circ \sigma$.

Entonces, de acuerdo con la observación 34.2, la definición 34.1, el teorema 34.1 y la regla de la cadena, para $s=\sigma(t)$ tenemos que:
\begin{align*}
\int_{\gamma_2} f(z) dz & = \int_{c}^{d} f(\gamma_2(t)) \gamma_2′(t)dt\\
& = \int_{c}^{d} f(\gamma_1(\sigma(t))) \gamma_1′(\sigma(t)) \sigma'(t)dt\\
& = \int_{\sigma(c)}^{\sigma(d)} f(\gamma_1(s)) \gamma_1′(s) ds\\
& = \int_{a}^{b} f(\gamma_1(s)) \gamma_1′(s) ds\\
& = \int_{\gamma_1} f(z) dz.
\end{align*}

$\blacksquare$

Ejemplo 34.5.
Determinemos el valor de la integral:
\begin{equation*}
\int_{C} \frac{dz}{z-2},
\end{equation*}donde $C$ es la semicircunferencia superior de radio $r=1$ y centro en $z_0=2$.

Solución. Es claro que $C$ es un contorno ya que la trayectoria $\gamma(t)=2+e^{i\pi}$, con $0\leq t\leq \pi$, que lo parametriza, es una curva suave.

De acuerdo con el ejemplo 34.1, inferimos que el valor de dicha integral es $i\pi$. Procedemos a verificar lo anterior utilizando la proposición 34.1, es decir, considerando otra parametrización para el contorno $C$.

De acuerdo con el ejemplo 32.13(b), sabemos que $\beta(t)=2+e^{i\pi t}$, con $0\leq t \leq 1$, es una reparametrización de la curva $\gamma$.

Entonces, por la definición 34.1 tenemos que:
\begin{align*}
\int_{C} \frac{1}{z-2} dz & =\int_{0}^{1} f(\beta(t)) \beta'(t)dt\\
& = \int_{0}^{1} \frac{1}{2 +e^{i\pi t} – 2} i\pi e^{i\pi t} dt\\
& = \int_{0}^{1} i\pi dt\\
& = i\pi.
\end{align*}

Al igual que con las integrales de funciones híbridas, las integrales de contorno cumplen algunas propiedades que resultan de utilidad al resolver ciertos problemas.

Proposición 34.2. (Propiedades integrales de contorno.)
Sean $U\subset\mathbb{C}$ un conjunto abierto, $[a,b]\subset\mathbb{R}$, con $a<b$ un intervalo cerrado, $f, g: U \to \mathbb{C}$ dos funciones continuas en $U$ y $\gamma:[a,b] \to U$ un contorno en $U$. Se cumplen las siguientes propiedades.

  1. Si $\lambda, \mu \in\mathbb{C}$ son dos constantes, entonces:
    \begin{equation*}
    \int_{\gamma} \left[\lambda f(z) + \beta g(z)\right] dz = \lambda \int_{\gamma} f(z) dz + \beta \int_{\gamma} g(z) dz.
    \end{equation*}
  2. \begin{equation*}
    \int_{-\gamma} f(z) dz = – \int_{\gamma} f(z) dz.
    \end{equation*}
  3. Si el contorno $\gamma$ es tal que $\gamma = \gamma_1 + \gamma_2$, entonces:
    \begin{equation*}
    \int_{\gamma} f(z) dz = \int_{\gamma_1} f(z) dz + \int_{\gamma_2} f(z) dz.
    \end{equation*}En general, si $\gamma = \gamma_1 + \cdots + \gamma_n$, entonces:
    \begin{equation*}
    \int_{\gamma} f(z) dz = \int_{\gamma_1} f(z) dz + \cdots + \int_{\gamma_n} f(z) dz = \sum_{k=1}^n \int_{\gamma_k} f(z) dz.
    \end{equation*}

Demostración. Dadas las hipótesis.

  1. Se deja como ejercicio al lector.
  2. De acuerdo con la definición 32.12 sabemos que $-\gamma(t) = \gamma(b+a-t)$ para toda $t\in[a,b]$, entonces, para $s=b+a-t$, por la observación 34.2, la definición 34.1, el teorema 34.1, la regla de la cadena y la proposición 33.1(7), tenemos que:
    \begin{align*}
    \int_{-\gamma} f(z) dz & = \int_{a}^{b} f(-\gamma(t)) \left[-\gamma(t)\right]’dt\\
    & = \int_{a}^{b} f(\gamma(a+b-t)) \left[-\gamma'(a+b-t)\right]dt\\
    & = \int_{b}^{a} f(\gamma(s)) \gamma'(s)ds\\
    & = -\int_{a}^{b} f(\gamma(s)) \gamma'(s)ds\\
    & = – \int_{\gamma} f(z) dz.
    \end{align*}
  3. Supongamos que existen $\gamma_1:[a_1, b_1] \to U$ y $\gamma_2:[a_2, b_2] \to U$ tales que $a=a_1$, $b=b_1+b_2-a_2$ y $\gamma_1(b_1)=\gamma_2(a_2)$, es decir, $\gamma=\gamma_1+\gamma_2$, entonces, por las definiciones 32.13, 34.1, la observación 34.2, la proposición 33.1(4) y el teorema 34.1, para $s=t-b_1+a_2$ tenemos que:
    \begin{align*}
    \int_{\gamma} f(z) dz & = \int_{\gamma_1+\gamma_2} f(z) dz\\
    & = \int_{a}^{b_1+b_2-a_2} f\left[\left(\gamma_1+\gamma_2\right)(t)\right] \left(\gamma_1+\gamma_2\right)'(t)dt\\
    & = \int_{a}^{b_1} f(\gamma_1(t)) \gamma_1′(t)dt + \int_{b_1}^{b_1+b_2-a_2} f(\gamma_2(t-b_1+a_2)) \gamma_2′(t-b_1+a_2)dt\\
    & = \int_{a}^{b_1} f(\gamma_1(t)) \gamma_1′(t)dt + \int_{a_2}^{b_2} f(\gamma_2(s)) \gamma_2′(s)ds\\
    & = \int_{\gamma_1} f(z) dz + \int_{\gamma_2} f(z) dz.
    \end{align*}El caso general se deja como ejercicio al lector.

$\blacksquare$

Observación 34.4.
Notemos que si $\gamma:[0,1] \to \mathbb{C}$ está dada por $\gamma(t)=it$ y $f(z)=1$, tenemos que:
\begin{align*}
\int_{\gamma} f(z) dz & = \int_{0}^{1} f(\gamma(t)) \gamma'(t)dt\\
& = \int_{0}^{1} 1 \cdot i \, dt\\
& = i.
\end{align*}

De donde se sigue que $\operatorname{Re}\left(\displaystyle \int_{\gamma} f(z) dz \right) = 0$.

Sin embargo, tenemos que $\operatorname{Re} f(z) = 1$, por lo que:
\begin{equation*}
\int_{\gamma} \operatorname{Re} f(z) dz = i.
\end{equation*}

Entonces, a diferencia de las integrales de funciones híbridas, para las integrales de contorno, en general tenemos que:
\begin{equation*}
\operatorname{Re}\left(\displaystyle \int_{\gamma} f(z) dz \right) \neq \int_{\gamma} \operatorname{Re} f(z) dz.
\end{equation*}

Ejemplo 34.6.
Verifiquemos el resultado del ejemplo 34.4 utilizando la proposición 34.2(3).

Solución. Para todo $t\in [0,1]$ se cumple que:
\begin{align*}
[0,1+i](t) & = (1+i)t, \quad [0,1+i]'(t) = 1+i,\\
[1+i,i](t) & = 1+i -t, \quad [1+i,i]'(t) = -1.
\end{align*}

Entonces, de acuerdo con la definición 34.1 y las proposiciones 33.2 y 34.2(3), tenemos que:
\begin{align*}
\int_{\gamma} (x+y) dz & = \int_{[0,1+i]} (x+y) dz + \int_{[1+i,i]} (x+y) dz\\
& = \int_{0}^{1} 2t(1+i) dt + \int_{0}^{1} (2-t)(-1) dt\\
&= \left.(1+i)t^2\right|_{0}^{1} + \left.\left[\frac{t^2}{2} – 2t\right]\right|_{0}^{1}\\
& = -\frac{1}{2} + i.
\end{align*}

Observación 34.5.
Aunque puede suceder que la integral de contorno de una función compleja a lo largo de dos curvas distintas sea la misma, esto en general no es cierto.

Ejemplo 34.7.
Veamos que:
\begin{equation*}
\int_{C_1} z dz = \int_{C_2} z dz,
\end{equation*}donde $C_1$ es el contorno dado por el segmento de recta que une a $z_1 = -1-i$ con $z_2 = 3+i$ y $C_2$ es el contorno que va de $z_1$ a $z_2$ a través del pedazo de la parábola $x=y^2+2y$, figura 123.

Figura 123: Contornos $C_1$ y $C_2$ del ejemplo 34.7.

Solución. De acuerdo con el ejemplo 32.2, podemos parametrizar al contorno $C_1$ mediante la trayectoria $\gamma_1:[0,1]\to\mathbb{C}$ dada por:
\begin{equation*}
\gamma_1(t) = -1-i+[3+i-(-1-i)]t = -1-i+(4+2i)t, \quad \forall t\in[0,1].
\end{equation*}

Por otra parte, podemos parametrizar al contorno $C_2$ como $\gamma_2(t)=x_2(t)+iy_2(t)$, donde:
\begin{equation*}
x_2(t)=t^2+2t, \quad y_2(t)=t.
\end{equation*}

Tenemos que $ t\in\mathbb{R}$, por lo que si $\gamma_2(t)=-1-i$, entonces:
\begin{equation*}
t^2+t(2+i)+1+i = (t+1)(t+1+i)=0 \quad \Longrightarrow \quad t=-1.
\end{equation*}

Análogamente, si $\gamma_2(t)=3+i$, entonces:
\begin{equation*}
t^2+t(2+i)-3-i = (t-1)(t+3+i)=0 \quad \Longrightarrow \quad t=1.
\end{equation*}

Por lo tanto $\gamma_2 : [-1,1]\to \mathbb{C}$, dada por $\gamma_2(t)=t^2+2t+it$, es una parametrización de $C_2$.

De acuerdo con la definición 34.1 y las proposiciones 33.1 y 33.2, tenemos que:
\begin{align*}
\int_{C_1} z dz & = \int_{0}^{1} \left[-1-i+(4+2i)t\right](4+2i) dt\\
& = (-1-i)(4+2i) \int_{0}^{1} dt + (4+2i)^2 \int_{0}^{1} t dt\\
& = (-2-6i) \left. t\right|_{0}^{1} + 4(3+4i) \left. \frac{t^2}{2}\right|_{0}^{1}\\
& = -2-6i + 6+8i\\
& = 4+2i.
\end{align*}
\begin{align*}
\int_{C_2} z dz & = \int_{-1}^{1} \left[t^2+2t+it\right](2t+2+i)dt\\
& = \int_{-1}^{1} \left[2t^3+6t^2+3t+i(3t^2+4t)\right]dt\\
& = \int_{-1}^{1} \left(2t^3+6t^2+3t\right)dt + +i\int_{-1}^{1}\left(3t^2+4t\right) dt\\
& = \left. \left(\frac{t^4}{2}+2t^3+\frac{3t^2}{2}\right)\right|_{-1}^{1} + \left. i\left(t^3+2t^2\right)\right|_{-1}^{1}\\
& = 4+2i.
\end{align*}

Ejemplo 34.8.
Veamos que:
\begin{equation*}
\int_{C_1} \overline{z} dz = -\pi i \quad \text{y} \quad \int_{C_2} \overline{z} dz = -4i,
\end{equation*}donde $C_1$ es el contorno que va de $-1$ a $1$ a través de la semicircunferencia unitaria superior y $C_2$ es el contorno que va de $-1$ a $1$ a través de la poligonal $[z_1, z_2, z_3, z_4]$, donde $z_1 = -1, z_2 = -1+i, z_3=1+i$ y $z_4 = 1$, ambos orientados negativamente, figura 124.

Solución. Considerando la definición 32.12, podemos parametrizar a $C_1$ mediante la curva opuesta de la semicircunferencia unitaria superior, orientada positivamente, es decir, $\beta(t)=e^{it}$, con $0\leq t \leq \pi$. Entonces, una parametrización del contorno $C_1$ está dada por la trayectoria $\gamma_1:[0,\pi]\to\mathbb{C}$ dada por:
\begin{equation*}
\gamma_1(t):= -\beta(t) = \beta(\pi + 0 – t) = \beta(\pi-t) = e^{i(\pi-t)} = -e^{-it}, \quad \forall t\in[0,\pi].
\end{equation*}

Considerando lo anterior, del ejemplo 32.1 se sigue que:
\begin{equation*}
\gamma_1′(t) = \frac{d}{dt} \left(-e^{-it}\right) = -(-i)e^{-it} = ie^{-it}.
\end{equation*}

Por otra parte, de acuerdo con la definición 32.13 y la observación 32.15, podemos parametrizar al contorno $C_2$, descrito por la poligonal $[z_1, z_2, z_3, z_4]$, donde $z_1 = -1, z_2 = -1+i, z_3=1+i$ y $z_4 = 1$, a través de la trayectoria $\gamma_2=[z_1, z_2] + [z_2, z_3] + [z_3, z_4]$. De acuerdo con el ejemplo 32.2 tenemos que:
\begin{align*}
[z_1, z_2](t) & = -1 + [-1+i-(-1)]t = -1+it,\\
[z_2, z_3](t) & = -1+i + [1+i-(-1+i)]t = -1+2t+i,\\
[z_3, z_4](t) & = 1+i + [1-(1+i)]t = 1+i(1-t),
\end{align*}donde $t\in[0,1]$ para los tres segmentos de recta. Entonces:
\begin{align*}
[z_1, z_2]'(t) & = \frac{d}{dt} \left(-1+it\right) = i,\\
[z_2, z_3]'(t) & = \frac{d}{dt} \left(-1+2t+i\right) = 2,\\
[z_3, z_4]'(t) & = \frac{d}{dt} \left(1+i(1-t)\right) = -i.
\end{align*}

Utilizando la definición 32.13 es fácil obtener de manera explícita la regla de correspondencia de $\gamma_2$, sin embargo, podemos utilizar la proposición 34.2(3) y simplificar las cuentas.

Figura 124: Contornos $C_1$ y $C_2$ del ejemplo 34.8.

Por lo tanto, de la definición 34.1 y las proposiciones 20.2 y 33.1, tenemos que:
\begin{align*}
\int_{C_1} \overline{z} dz & = \int_{0}^{\pi} \overline{-e^{-it}}(ie^{-it}) dt\\
& = -i\int_{0}^{\pi}e^{it}e^{-it} dt\\
& = -i\int_{0}^{\pi} e^{0} dt\\
& = -i \pi.
\end{align*}

Mientras que de la definición 34.1 y las proposiciones 34.2(3), 33.1 y 33.2, se sigue que:
\begin{align*}
\int_{C_2} \overline{z} dz & = \int_{ [z_1, z_2]} \overline{z} dz + \int_{ [z_2, z_3]} \overline{z} dz + \int_{ [z_3, z_4]} \overline{z} dz\\
& = \int_{0}^{1} \left(\overline{-1+it}\right) i dt + \int_{0}^{1} \left(\overline{-1+2t+i}\right) 2 dt + \int_{0}^{1} \left[\overline{1+i(1-t)}\right](-i) dt\\
& = i\int_{0}^{1} \left(-1-it\right) dt + 2 \int_{0}^{1} \left(-1+2t-i\right) dt -i \int_{0}^{1} \left[1-i(1-t)\right] dt\\
& = \left.i\left(-t-i\frac{t^2}{2}\right)\right|_{0}^{1} + \left. 2 \left(-t+t^2-it\right)\right|_{0}^{1} – \left. i \left[t+i\frac{(1-t)^2}{2}\right]\right|_{0}^{1}\\
& = i\left(-1-\frac{i}{2}\right) – 2i – i \left(1-\frac{i}{2}\right)\\
& = -4i.
\end{align*}

Definición 34.2. (Integral con respecto de la longitud de arco.)
Sean $U\subset\mathbb{C}$ un conjunto abierto, $[a,b]\subset\mathbb{R}$, con $a<b$ un intervalo cerrado, $f: U \to \mathbb{C}$ una función continua en $U$ y $\gamma:[a,b] \to U$ un contorno en $U$. Se define a la integral de $f$ {\bf con respecto de la longitud de arco $|dz|$}, a lo largo de $\gamma$, como:
\begin{equation*}
\int_{\gamma} f(z) |dz| := \int_{a}^{b} f(\gamma(t)) \left|\gamma'(t)\right| dt.\tag{34.3}
\end{equation*}

Si el contorno está dado por una trayectoria $\gamma$ suave a trozos, para la partición:
\begin{equation*}
P : a=t_0 < t_1 < \cdots < t_{n-1}<t_n=b,
\end{equation*}del intervalo $[a,b]$, tal que $\gamma_k$, la restricción de $\gamma$ al intervalo $[t_{k-1}, t_k]$, es una curva suave para $1\leq k\leq n$, se cumple que:
\begin{equation*}
\int_{\gamma} f(z) |dz| = \int_{\gamma_1} f(z) |dz| + \cdots + \int_{\gamma_n} f(z) |dz| = \sum_{k=1}^n \int_{\gamma_k} f(z) |dz|. \tag{34.4}
\end{equation*}

Observación 34.6.
Notemos que si $f(z)=1$, entonces de (34.3) obtenemos:
\begin{equation*}
\int_{\gamma}|dz| = \int_{a}^{b} \left|\gamma'(t)\right| dt,
\end{equation*}la cual corresponde con la longitud de arco de una curva en $\mathbb{C}$, definición 32.15.

El siguiente resultado justifica la definición anterior.
Lema 34.1.
Si $\gamma$ es una curva suave a trozos, es decir, un contorno en $\mathbb{C}$, entonces $\gamma$ es rectificable (definición 32.16) y la longitud de arco de dicha curva es:
\begin{equation*}
\ell(\gamma)=\int_{\gamma}|dz|.
\end{equation*}

Se puede consultar una prueba detallada de este resultado en:

  • An Introduction to Complex Function Theory, Bruce P. Palka.
  • Function of One Complex Variable, John B. Conway.
  • Teoría de funciones de una variable compleja, Felipe Zaldívar.

Ejemplo 34.9.
Evaluemos las siguientes integrales.
a) $\displaystyle \int_{\gamma} z^{-2} |dz|$, donde la trayectoria $\gamma$ describe a la circunferencia $C(0,2)$ orientada positivamente, es decir, $\gamma(t)=2e^{it}$, con $0\leq t\leq 2\pi$.
b) $\displaystyle \int_{\gamma} x |dz|$, donde $\gamma(t)=t+i\left(\dfrac{t^2}{2}\right)$, con $0\leq t\leq 1$, figura 125.

Figura 125: Contorno $\gamma(t)=t+i\left(\dfrac{t^2}{2}\right)$, con $0\leq t\leq 1$.

Solución.

a) Es claro que $\gamma$ es un contorno y $\gamma'(t)=i2e^{it}$. Más aún, sabemos que la función $f(z)=z^{-2}$ es analítica en el dominio $D=\mathbb{C}\setminus\{0\}$, por lo que es continua en $D$ y el contorno descrito por $\gamma$ está completamente contenido en $D$. Entonces, por la definción 34.2, las proposiciones 20.2(6), 20.2(7), 33.1(3) y el ejemplo 33.2, tenemos que:
\begin{align*}
\int_{\gamma} z^{-2} |dz| & = \int_{0}^{2\pi} \left(2e^{it}\right)^{-2}\left|i2e^{it}\right| dt\\
& = \frac{1}{2} \int_{0}^{2\pi}e^{-i2t} dt\\
& = \frac{1}{2}(0)\\
& = 0.
\end{align*}

b) Es claro que $\gamma$ es un contorno, con $\gamma'(t)=1+it$. Por otra parte, por el ejemplo 15.1(a) sabemos que la función $f(z)=\operatorname{Re}(z)=x$, para $z=x+iy\in\mathbb{C}$, es continua en todo $\mathbb{C}$. Entonces, por la definción 34.2 tenemos que:
\begin{align*}
\int_{\gamma} x |dz| & = \int_{0}^{1} t\left|1+t^2\right| dt\\
& = \frac{1}{2} \int_{0}^{1}2t\sqrt{1+t^2}dt\\
& = \frac{1}{2} \left.\left[ \frac{2\left(1+t^2\right)^{3/2}}{3}\right]\right|_{0}^{1}\\
& = \frac{2\sqrt{2}-1}{3}.
\end{align*}

Proposición 34.3. (Propiedades integrales con respecto de la longitud de arco.)
Sean $U\subset\mathbb{C}$ un conjunto abierto, $[a,b]\subset\mathbb{R}$, con $a<b$ un intervalo cerrado, $f, g: U \to \mathbb{C}$ dos funciones continuas en $U$ y $\gamma:[a,b] \to U$ un contorno en $U$. Se cumplen las siguientes propiedades.

  1. Si $\lambda, \mu \in\mathbb{C}$ son dos constantes, entonces:
    \begin{equation*}
    \int_{\gamma} \left[\lambda f(z) + \beta g(z)\right] |dz| = \lambda \int_{\gamma} f(z) |dz| + \beta \int_{\gamma} g(z) |dz|.
    \end{equation*}
  2. \begin{equation*}
    \int_{-\gamma} f(z) |dz| = \int_{\gamma} f(z) |dz|.
    \end{equation*}
  3. Si el contorno $\gamma$ es tal que $\gamma = \gamma_1 + \gamma_2$, entonces:
    \begin{equation*}
    \int_{\gamma} f(z) |dz| = \int_{\gamma_1} f(z) |dz| + \int_{\gamma_2} f(z) |dz|.
    \end{equation*}En general, si $\gamma = \gamma_1 + \cdots + \gamma_n$, entonces:
    \begin{equation*}
    \int_{\gamma} f(z) |dz| = \int_{\gamma_1} f(z) |dz| + \cdots + \int_{\gamma_n} f(z) |dz| = \sum_{k=1}^n \int_{\gamma_k} f(z) |dz|.
    \end{equation*}
  4. Si $\beta$ es una reparametrización de $\gamma$, entonces:
    \begin{equation*}
    \int_{\beta} f(z) |dz| = \int_{\gamma} f(z) |dz|.
    \end{equation*}
  5. \begin{equation*}
    \left|\int_{\gamma} f(z) dz \right| \leq \int_{\gamma} |f(z)| |dz|.
    \end{equation*}En particular, si $M$ es una constante tal que $|f(z)|\leq M$ y $L=\ell\left(\gamma\right)$, entonces:
    \begin{equation*}
    \left|\int_{\gamma} f(z) dz\right| \leq ML.
    \end{equation*}

Demostración. Dadas las hipótesis.

  1. Se deja como ejercicio al lector.
  2. Se deja como ejercicio al lector.
  3. Se deja como ejercicio al lector.
  4. Se deja como ejercicio al lector.
  5. De acuerdo con la definición 34.1 y la proposición 33.1(5) tenemos que:
    \begin{align*}
    \left|\int_{\gamma} f(z) dz \right| & = \left| \int_{a}^{b} f(\gamma(t)) \gamma'(t) dt\right|\\
    & \leq \int_{a}^{b} \left| f(\gamma(t))\gamma'(t)\right| dt\\
    & = \int_{a}^{b} \left| f(\gamma(t))\right| \, \left|\gamma'(t)\right| dt\\
    & = \int_{\gamma} |f(z)| |dz|.
    \end{align*}Si $M=\max\limits_{z\in\gamma}|f(z)|$ y $L=\ell\left(\gamma\right)$, entonces $|f(z)| \leq M$, por lo que de la monotonía de la integral para funciones reales se sigue que:
    \begin{equation*}
    \left|\int_{\gamma} f(z) dz \right| \leq \int_{\gamma} |f(z)| |dz| \leq \int_{a}^{b} M |\gamma'(t)| dt = ML.
    \end{equation*}

$\blacksquare$

Observación 34.7.
Muchas veces, en la teoría y en la práctica, no es necesario evaluar una integral de contorno, sino que simplemente basta con obtener una cota superior de su módulo, por ello la propiedad dada en la proposición 34.3(5) es de mucha utilidad.

Ejemplo 34.10.
Determinemos una cota superior para:
\begin{equation*}
\left|\int_{\gamma} \frac{e^{z}}{z^2+1} dz \right|,
\end{equation*}donde $\gamma$ describe a la circunferencia $C(0,2)$ en sentido positivo.

Solución. Tenemos que una parametrización del contorno $C(0,2)$ es $\gamma(t)=2e^{it}$, para $0\leq t \leq 2\pi$. Sabemos que $\gamma'(2)=i2e^{it}$, entonces, de la proposición 20.2(6) y la definción 32.15 se sigue que:
\begin{equation*}
L := \ell(\gamma) = \int_{0}^{2\pi}|\gamma'(t)| dt = \int_{0}^{2\pi}|i2e^{it}| dt = \int_{0}^{2\pi} 2 dt = 4\pi.
\end{equation*}

Por el corolario 16.1(2) es claro que la función racional:
\begin{equation*}
f(z) = \frac{e^{z}}{z^2+1},
\end{equation*}es analítica en $D=\mathbb{C}\setminus\{-i, i\}$ y por tanto continua en $D$. Además el contorno $C(0,2)$ está completamente contenido en $D$.

Por la proposición 20.2(4), para $z=x+iy\in\mathbb{C}$ sabemos que $|e^z| = e^x$ y de la observación 3.1 tenemos que $x = \operatorname{Re}(z)\leq |z|$, entonces, corolario 31.1(1), $|e^z| \leq e^{|z|}$.

De lo anterior, para $z=\gamma(t)$ tenemos que:
\begin{equation*}
\left|e^{2e^{it}}\right| \leq e^{|2e^{it}|} = e^{2},
\end{equation*}y considerando la desigualdad del triángulo, proposición 3.3, tenemos que:
\begin{equation*}
\left|z^2+1\right| = \left|\left(2e^{it}\right)^2+1\right| = \left|4e^{i2t}+1\right| \geq \left|4e^{i2t}\right| – \left|1\right| = 4-1=3.
\end{equation*}

Entonces, para $z=\gamma(t)$, es decir, para $|z|=2$, se cumple que:
\begin{equation*}
\left|f(z)\right| = \left|\frac{e^{z}}{z^2+1}\right| \leq \frac{e^2}{3} =: M.
\end{equation*}

Por lo tanto, por la proposición 34.3(5) tenemos que:
\begin{equation*}
\left|\int_{\gamma} \frac{e^{z}}{z^2+1} dz \right| \leq M L = \frac{4\pi e^2}{3}.
\end{equation*}

Ejemplo 34.11.
Sea $r>0$. Veamos que:
\begin{equation*}
\left|\int_{\gamma} e^{iz^2} dz \right| \leq \frac{\pi(1-e^{-r^2})}{4r},
\end{equation*}donde $\gamma(t) = re^{it}$, con $0\leq t \leq \dfrac{\pi}{4}$.

Solución. Sabemos que la función $f(z)=e^{iz^2}$ es entera y por tanto continua en $\mathbb{C}$. Por otra parte, es claro que el arco de circunferencia $C(0,r)$ descrito por $\gamma(t) = re^{it}$, $0\leq t \leq \dfrac{\pi}{4}$, es un contorno en $\mathbb{C}$ y $\gamma'(t)=ire^{it}$. Entonces, por la proposición 20.2(6), es claro que:
\begin{equation*}
\left|\gamma'(t)\right| = \left|ire^{it}\right| = r.
\end{equation*}

Si $z=x+iy\in\mathbb{C}$, entonces $iz^2= i(x^2-y^2) – 2xy$, por lo que, de acuerdo con la proposición 20.2(4), tenemos que:
\begin{equation*}
\left|f(z)\right| = \left|e^{iz^2}\right| = e^{\operatorname{Re}\left(iz^2\right)} = e^{-2xy}.
\end{equation*}

De la proposición 20.2(5) se sigue que $\gamma(t)=re^{it} = r\operatorname{cos}(t)+ir\operatorname{cos}(t)$, entonces:
\begin{equation*}
\left|f\left(\gamma(t)\right)\right| = e^{-2r^2\operatorname{cos}(t)\operatorname{sen}(t)} = e^{-r^2\operatorname{sen}(2t)}.
\end{equation*}

Sea $u=2t$. Notemos que:
\begin{equation*}
0\leq u \leq \dfrac{\pi}{2} \quad \Longrightarrow \quad \operatorname{sen}(u) \geq \frac{2u}{\pi} \quad \Longrightarrow \quad -r^2 \operatorname{sen}(u) \leq -\frac{2ur^2}{\pi}.
\end{equation*}

Por lo que:
\begin{align*}
\int_{\gamma} \left|f(z)\right| |dz| & = \int_{0}^{\pi/4} \left|f\left(\gamma(t)\right)\right| \, \left|\gamma'(t)\right| dt\\
& = \int_{0}^{\pi/4} r e^{-r^2\operatorname{sen}(2t)} dt\\
& = \frac{r}{2} \int_{0}^{\pi/2} e^{-r^2\operatorname{sen}(u)} du\\
& \leq \frac{r}{2} \int_{0}^{\pi/2} \operatorname{exp}\left(-\dfrac{2ur^2}{\pi}\right) du\\
& = \left.\left[-\dfrac{\pi \operatorname{exp}\left(-\dfrac{u2r^2}{\pi}\right)}{4r}\right]\right|_{0}^{\pi/2}\\
& = \frac{\pi(1-e^{-r^2})}{4r}.
\end{align*}

Entonces, por la proposición 34.3(5) tenemos que:
\begin{equation*}
\left|\int_{\gamma} e^{iz^2} dz \right| \leq \int_{\gamma} \left|e^{iz^2}\right| \, |dz| \leq \frac{\pi(1-e^{-r^2})}{4r}.
\end{equation*}

Observación 34.8.
En este punto es importante hacer un comentario sobre la notación para integrales de contorno a lo largo de segmentos de recta. Si $f$ es una función compleja continua en el segmento de recta que une a los puntos $z_1, z_2\in\mathbb{C}$, con $z_1\neq z_2$, es decir, $f$ es continua en $[z_1, z_2]$, entonces denotamos lo anterior como:
\begin{equation*}
\int_{[z_1, z_2]} f(z) dz := \int_{z_1}^{z_2} f(z) dz.
\end{equation*}
\begin{equation*}
\int_{[z_1, z_2]} f(z) |dz| := \int_{z_1}^{z_2} f(z) |dz|.
\end{equation*}

Así por ejemplo, como $[z_2, z_1](t) = -[z_1, z_2](t)$, de la proposición 34.2(2) se sigue que:
\begin{equation*}
\int_{z_2}^{z_1} f(z) dz = – \int_{z_1}^{z_2} f(z) dz.
\end{equation*}

Además, como $[z_1, z_1]$ corresponde con un contorno constante, entonces:
\begin{equation*}
\int_{z_1}^{z_1} f(z) dz = 0.
\end{equation*}

Considerando lo anterior, si $z_3$ es un tercer punto en el segmento $[z_1, z_2]$, distinto de $z_1$ y de $z_2$, entonces:
\begin{equation*}
\int_{z_1}^{z_2} f(z) dz = \int_{z_1}^{z_3} f(z) dz + \int_{z_3}^{z_2} f(z) dz.
\end{equation*}

Debe ser claro que lo anterior no es una consecuencia directa de la proposición 34.2(3), ya que si consideramos la definición 32.13, no es difícil verificar que el contorno dado por $[z_1, z_3] + [z_3, z_2]$ no es igual al contorno dado por $[z_1, z_2]$.

Ejemplo 34.12.
Si $R\subset\mathbb{C}$ es un rectángulo en el plano complejo con vértices $z_1, z_2, z_3, z_4\in\mathbb{C}$, entonces el contorno poligonal dado por $\gamma = [z_1, z_2] + [z_2, z_3] + [z_3, z_4] + [z_4, z_1]$ parametriza a la frontera $\partial R$ de dicho rectángulo, en sentido positivo relativo a $R$, figura 126. Considerando la notación dada en la observación 34.6, la integral de contorno de una función $f$ continua a lo largo de $\gamma$ está dada por:
\begin{equation*}
\int_{\delta R} f(z) dz = \int_{z_1}^{z_2} f(z) dz + \int_{z_2}^{z_3} f(z) dz + \int_{z_3}^{z_4} f(z) dz + \int_{z_4}^{z_1} f(z) dz.
\end{equation*}

Figura 126: Rectángulo $R$ en el plano complejo $\mathbb{C}$ y su frontera $\partial R$.

Tarea moral

  1. Completa las demostraciones de las proposiciones 34.2 y 34.3.
  2. Evalúa las siguientes integrales.
    a) $\displaystyle \int_{\gamma} (2xy-ix^2) dz$, donde $\gamma(t)=t+it^2$, con $0\leq t\leq 1$.
    b) $\displaystyle \int_{\gamma} \dfrac{z^2-1}{z(z^2+4)} dz$, donde $\gamma(t)=e^{it}$, con $0\leq t\leq 2\pi$.
    Hint: Utiliza fracciones parciales.
    c) $\displaystyle \int_{C} z^2 |dz|$, donde $C=C(i,2)$, orientada positivamente.
    d) $\displaystyle \int_{\gamma} z |dz|$, donde $C\gamma=[e, 1] + [1, -1+i\sqrt{3}]$.
  3. Sea $C$ el contorno dado por el segmento de recta que va de $1$ a $i$. Determina una cota superior para:
    \begin{equation*}
    \left|\int_{C} \operatorname{cos}^2(z) dz\right|.
    \end{equation*}
  4. Sea $f:C(0,1) \to \mathbb{C}$ una función continua tal que $|f(z)|\leq M$ para todo $z\in C(0,1)$, con $M>0$. Prueba que si:
    \begin{equation*}
    \left|\int_{C(0,1)} f(z) dz\right| = 2\pi M,
    \end{equation*}
    entonces $f(z)=c\overline{z}$, donde $c\in\mathbb{C}$ es una constante tal que $|c|=M$.
    Hint: Considera el ejercicio 4 de la entrada 33.
  5. Si $\gamma(t)=e^{1+it}$, con $0\leq t \leq \pi$, muestra que:
    \begin{equation*}
    \left|\int_{\gamma} \left[\operatorname{Log}(z)\right]^{-1} dz\right| \leq e \operatorname{Log}(\pi+\sqrt{\pi^2+1}).
    \end{equation*}
  6. Sean $P(z)$ y $Q(z)$ dos polinomios complejos de grado $n$ y $m$, respectivamente, tales que $m\geq n+2$. Muestra que:
    \begin{equation*}
    \lim\limits_{r\to \infty} \int_{C} \frac{P(z)}{Q(z)} dz = 0,
    \end{equation*}donde el contorno $C$ es la circunferencia $C(0,r)$.
    Hint: Utiliza la proposición 34.3(5).
  7. Evalúa la integral $\int_{\gamma} \overline{z} dz$, donde:
    a) $\gamma$ es el pedazo de la parábola $y=x^2$ que va de $0$ a $1+i$;
    b) $\gamma$ es el arco de la cicloide dada por:
    \begin{equation*}
    x(t)=a(t-\operatorname{sen}(t)), \quad y(t)=a(1-\operatorname{cos}(t)),
    \end{equation*}entre los puntos $(0,0)$ y $(a\pi, 2a)$, con $a>0$.
  8. Verifica que:
    \begin{equation*}
    \int_{\gamma_1} \frac{1}{z} dz \neq \int_{\gamma_2} \frac{1}{z} dz,
    \end{equation*}donde $\gamma_1(t)=e^{-it}$ y $\gamma_2(t)=e^{it}$, con $t\in[0,2\pi]$.

Más adelante…

En esta entrada hemos definido de manera formal lo que es una integral de una función compleja de variable compleja. Como vimos, esta definición es similar a la de una integral de línea y muchos de las propiedades de este tipo de integrales están sustentados por la teoría de integración para integrales reales, por lo que la operabilidad de estas integrales resulta sencilla gracias a los resultados de nuestros cursos de Cálculo.

En la siguiente entrada probaremos el Teorema Fundamental del Cálculo para integrales de contorno y el lema de Goursat, así como otros resultados importantes sobre las integrales de contorno para funciones complejas, los cuales nos serán de utilidad para probar algunos de los resultados fundamentales en la teoría de la Variable Compleja, como el teorema de Cauchy.

Entradas relacionadas

Variable Compleja I: Integrales de funciones híbridas

Por Pedro Rivera Herrera

Introducción

En esta entrada veremos algunos de los conceptos básicos, pero elementales, de las integrales para funciones complejas de variable real. Para ello recurriremos a algunos resultados de nuestros cursos de Cálculo.

Primeramente consideremos a una función híbrida $f(t)=u(t)+iv(t)$, con $t\in[a,b]\subset\mathbb{R}$ y $a<b$. Tenemos que $u(t)$ y $v(t)$ son ambas funciones reales de variable real. De acuerdo con nuestros cursos de Cálculo, sabemos que si $u$ y $v$ son funciones continuas en el intervalo $[a,b]$, entonces ambas son funciones Riemann-integrables para la variable $t$, es decir, las integrales de Riemann $\int_{a}^{b} u(t) dt$ y $\int_{a}^{b} v(t) dt$ existen. Considerando lo anterior tenemos la siguiente:

Definición 33.1. (Integral compleja de una función híbrida.)
Sean $[a,b]\subset{\mathbb{R}}$ un intervalo cerrado, con $a<b$, y $f: [a,b] \to \mathbb{C}$ una función híbrida continua en $[a,b]$. Para $f(t) = u(t) + i v(t)$ se define a la integral de $f$ en $[a,b]$ como:
\begin{equation*}
\int_{a}^{b} f(t) \, dt
:= \int_{a}^{b} u(t)\, dt + i \int_{a}^{b} \,v(t) dt.
\end{equation*}

Es decir, $\int_{a}^{b} f(t) \,dt$ existe si y solo si $\int_{a}^{b} u(t) \,dt$ y $\int_{a}^{b} v(t) \,dt$ existen, en tal caso se dice que $f$ es integrable.

Observación 33.1.
Por nuestros cursos de Cálculo sabemos que una función real que es continua por partes o a trozos también es Riemann-integrable, por lo que, considerando la definición 32.3, podemos extender la definición 33.1 para funciones híbridas que son continuas a trozos.

Definición 33.2. (Integral compleja de una función híbrida a trozos.)
Sean $[a,b]\subset{\mathbb{R}}$ un intervalo cerrado, con $a<b$, y $f: [a,b] \to \mathbb{C}$ una función híbrida continua a trozos en $[a,b]$. Para la partición:
\begin{equation*}
P : a=t_0 < t_1 < \cdots < t_{n-1}<t_n=b,
\end{equation*}del intervalo $[a,b]$, se define a la integral de $f$ en $[a,b]$ como:
\begin{equation*}
\int_{a}^{b} f(t) \,dt
= \displaystyle\sum_{k=1}^n \int_{t_k}^{t_{k-1}} f(t) \,dt.
\end{equation*}

Observación 33.2.
Recordemos que no es esencial que la función $f$ esté definida en los puntos $t_0, t_1, \ldots, t_n$ ya que el valor de $f$ en dicho conjunto finito de puntos se puede asignar o cambiar de forma arbitraria sin afectar el valor de la integral.

Debe ser claro que las integrales complejas de este tipo heredan todas las propiedades de la integral de funciones reales de variable real.

Proposición 33.1.
Sean $[a,b]\subset{\mathbb{R}}$ un intervalo cerrado, con $a<b$, $f, g: [a,b] \to \mathbb{C}$ dos funciones híbridas continuas en $[a,b]$ y sea $k\in\mathbb{C}$ una constante. Se satisfacen las siguientes propiedades.

  1. \begin{equation*}
    \operatorname{Re}\left( \int_{a}^{b} f(t) \,dt\right) = \int_{a}^{b} \operatorname{Re} f(t) \,dt \quad \text{e} \quad \operatorname{Im}\left( \int_{a}^{b} f(t) \,dt\right) = \int_{a}^{b} \operatorname{Im} f(t) \,dt.
    \end{equation*}
  2. \begin{equation*}
    \int_{a}^{b} \left[f(t) \pm g(t) \right]\,dt = \int_{a}^{b} f(t)\,dt \pm \int_{a}^{b} g(t)\,dt.
    \end{equation*}
  3. \begin{equation*}
    \int_{a}^{b} kf(t)\,dt = k\int_{a}^{b} f(t)\,dt.
    \end{equation*}
  4. Si $c\in(a,b)$, entonces:
    \begin{equation*}
    \int_{a}^{b} f(t)\,dt = \int_{a}^{c} f(t)\,dt + \int_{c}^{b} f(t)\,dt.
    \end{equation*}
  5. \begin{equation*}
    \left|\int_{a}^{b} f(t)\,dt\right| \leq \int_{a}^{b}\left| f(t) \right| \,dt
    \end{equation*}
  6. Si $f$ y $g$ son diferenciables en $(a,b)$ y continuas en $[a,b]$, entonces:
    \begin{equation*}
    \int_{a}^{b} f(t) g'(t)\,dt = f(b)g(b) – f(a)g(a) – \int_{a}^{b} f'(t) g(t)\,dt,
    \end{equation*}es decir, la integración por partes se cumple para funciones híbridas.
  7. \begin{equation*}
    \int_{b}^{a} f(t)\,dt = -\int_{a}^{b} f(t)\,dt.
    \end{equation*}

Demostración. Dadas las hipótesis.

  1. Es inmediata de la definición 33.1, por lo que los detalles se dejan como ejercicio al lector.
  2. Se deja como ejercicio al lector.
  3. Sean $f(t)=u(t)+iv(t)$ y $k=\alpha+i\beta$, con $\alpha, \beta\in\mathbb{R}$. Para toda $t\in[a,b]$ tenemos que:
    \begin{align*}
    k f(t) &= (\alpha+i\beta)(u(t)+iv(t))\\
    &= \alpha u(t)- \beta v(t) + i\left[\alpha v(t) +i\beta u(t)\right].
    \end{align*}Entonces, de la definición 33.1 y aplicando las propiedades de linealidad de las integrales de funciones reales, tenemos que:
    \begin{align*}
    \int_{a}^{b} k f(t) \, dt &= \int_{a}^{b} \left[\alpha u(t) – \beta v(t)\right] \, dt + i \int_{a}^{b} \left[\alpha v(t) +i\beta u(t)\right] \, dt\\
    &= \alpha \int_{a}^{b} u(t) dt – \beta \int_{a}^{b} v(t) dt + i \left[\alpha \int_{a}^{b} v(t) dt +\beta \int_{a}^{b} u(t) dt\right]\\
    & = (\alpha + i\beta) \left[\int_{a}^{b} u(t) dt + i \int_{a}^{b} v(t) dt\right]\\
    & = k \int_{a}^{b} f(t) \, dt.
    \end{align*}
  4. Se deja como ejercicio al lector.
  5. Si $\displaystyle\int_{a}^{b} f(t)\,dt = 0$, entonces:
    \begin{equation*}
    \left|\int_{a}^{b} f(t)\,dt\right| = 0 \leq \int_{a}^{b}\left| f(t) \right| \,dt,
    \end{equation*}por lo que en tal caso no hay nada que probar.

    Supongamos que $\displaystyle\int_{a}^{b} f(t)\,dt \neq 0$, entonces podemos escribir a la integral en su forma polar, es decir:
    \begin{equation*}
    \int_{a}^{b} f(t)\,dt = r e^{i\theta},
    \end{equation*}donde $r=\left|\int_{a}^{b} f(t)\,dt\right|\geq 0$ y $\theta = \operatorname{arg}\left(\int_{a}^{b} f(t)\,dt\right)$.

    Considerando lo anterior y la propiedad 3 tenemos que:
    \begin{equation*}
    r = \left|\int_{a}^{b} f(t)\,dt\right| = e^{-i\theta} \int_{a}^{b} f(t)\,dt = \int_{a}^{b} e^{-i\theta} f(t)\,dt.
    \end{equation*}Como las cantidades de la igualdad anterior son números reales, tomando la parte real de ambos lados de la igualdad, de la propiedad 1 se sigue que:
    \begin{equation*}
    \left|\int_{a}^{b} f(t)\,dt\right| = \operatorname{Re} \left(\int_{a}^{b} e^{-i\theta} f(t)\,dt\right) = \int_{a}^{b} \operatorname{Re} \left(e^{-i\theta} f(t)\right) \,dt.
    \end{equation*}Recordemos que para todo $z\in\mathbb{C}$ se cumple que $\operatorname{Re}(z) \leq |z|$, por lo que, considerando la monotonía de la integral para funciones reales y la proposición 20.2, tenemos que:
    \begin{align*}
    \left|\int_{a}^{b} f(t)\,dt\right| & = \int_{a}^{b} \operatorname{Re} \left(e^{-i\theta} f(t)\right) \,dt\\
    & \leq \int_{a}^{b} \left|e^{-i\theta} f(t)\right| \,dt\\
    & = \int_{a}^{b} \left|e^{-i\theta}\right| \left|f(t)\right| \,dt\\
    & = \int_{a}^{b} \left|f(t)\right| \,dt.
    \end{align*}Notemos que el resultado se cumple sin importar la rama del argumento que elijamos.
  6. Se sigue de desarrollar el producto de $f(t)$ y $g'(t)$ y aplicar integración por partes para funciones reales, por lo que los detalles se dejan como ejercicio al lector.
  7. Se deja como ejercicio al lector.

$\blacksquare$

Observación 33.3.
Notemos que si $M=\sup\limits_{t\in[a,b]} |f(t)| < \infty$, entonces se cumple que:
\begin{equation*}
\left|\int_{a}^{b} f(t)\,dt\right| \leq \int_{a}^{b}\left| f(t) \right| \,dt \leq \int_{a}^{b} M \,dt = M(b-a).
\end{equation*}

Ejemplo 33.1.
Obtengamos la integral $\displaystyle\int_{0}^{2} f(t)\,dt$, donde:
\begin{equation*}
f(t)= \left\{ \begin{array}{lcc}
(1+i)t& \text{si} & 0\leq t \leq 1, \\ \\
it^2 & \text{si} & 1\leq t \leq 2.
\end{array} \right.
\end{equation*}

Solución. De acuerdo con la proposición 33.1(3) y 33.1(4) tenemos que:
\begin{align*}
\int_{0}^{2} f(t)\,dt &= \int_{0}^{1} f(t)\,dt + \int_{1}^{2} f(t)\,dt\\
&= (1+i)\int_{0}^{1} t\,dt + i \int_{1}^{2} t^2\,dt\\
& = \frac{(1+i)(1^2-0^2)}{2} + \frac{i(2^3-1^3)}{3}\\
& = \frac{1}{2} + i\frac{17}{6}.
\end{align*}

Definición 33.2. (Primitiva de una función híbrida.)
Sean $[a,b]\subset{\mathbb{R}}$ un intervalo cerrado, con $a<b$, y $f: [a,b] \to \mathbb{C}$ una función híbrida continua en $[a,b]$. Si existe una función continua $F: [a,b] \to \mathbb{C}$ tal que:
\begin{equation*}
F'(t)=f(t), \quad \forall t\in(a,b),
\end{equation*}se dice que $F$ es una primitiva de $f$.

Observación 33.4.
Debe ser claro que si $f, F: [a,b] \to \mathbb{C}$ son dos funciones híbridas continuas en $[a,b]$, tales que:
\begin{equation*}
f(t)=u(t) +iv(t) \quad \text{y} \quad F(t)=U(t) +iV(t),
\end{equation*}entonces $F$ es primitiva de $f$ si y solo si $U$ es primitiva de $u$ y $V$ es primitiva de $v$, es decir, las funciones reales $U(t)$ y $V(t)$ son tales que $U'(t)=u(t)$ y $V'(t)=v(t)$.

Veamos que para las funciones híbridas el segundo Teorema Fundamental del Cálculo (TFC), es válido.

Proposición 33.2. (Segundo TFC para funciones híbridas.)
Sean $[a,b]\subset{\mathbb{R}}$ un intervalo cerrado, con $a<b$, y $f: [a,b] \to \mathbb{C}$ una función híbrida continua en $[a,b]$. Si $F: [a,b] \to \mathbb{C}$ es una primitiva de $f$, entonces:
\begin{equation*}
\int_{a}^{b} f(t)\,dt = \left. F(t) \right|_{a}^{b}
= F(b) – F(a).
\end{equation*}

Demostración. Dadas las hipótesis, sean $f(t)=u(t)+iv(t)$ y $F(t)=U(t)+iV(t)$. Dado que $F$ es una primitiva de $f$, entonces, por la observación 33.4 y considerando el segundo TFC para funciones reales, tenemos que:
\begin{align*}
\int_{a}^{b} f(t) dt & = \int_{a}^{b} u(t)\, dt + i \int_{a}^{b} \,v(t) dt\\
& = \left[U(b)-U(a)\right] + i \left[V(b)-V(a)\right]\\
& = \left[U(b)+iV(b)\right] -\left[U(a)+iV(a)\right]\\
& = F(b) – F(a).
\end{align*}

En el caso en que $f$ es continua a trozos en $[a,b]$, podemos tomar por definición a la partición:
\begin{equation*}
P : a=t_0 < t_1 < \cdots < t_{n-1}<t_n=b,
\end{equation*}del intervalo $[a,b]$, donde $t_1, \ldots, t_{n-1}$ son los puntos de discontinuidad de la función continua a trozos $f$ en $(a,b)$. Entonces por la proposición 33.1(4) tenemos que:
\begin{align*}
\int_{a}^{b} f(t) dt & = \displaystyle\sum_{k=1}^n \int_{t_k}^{t_{k-1}} f(t) \,dt\\
& = \displaystyle\sum_{k=1}^n \left[ F(t_k) – F(t_{k-1})\right]\\
& = F(t_n) – F(t_{0})\\
& = F(b) – F(a).
\end{align*}

$\blacksquare$

Observación 33.5.
Por simplicidad hemos enunciado los resultados anteriores para funciones híbridas continuas, sin embargo, tanto las definiciones anteriores como las propiedades de la proposición 33.1 y el segundo TFC, para funciones híbridas, siguen siendo válidos si $f$ y $g$ son funciones continuas a trozos en $[a,b]$, con una adecuada modificación de los enunciados considerando los resultados de la teoría de integración para funciones reales continuas a trozos y la definición 33.2.

Ejemplo 33.2.
Sea $n\in\mathbb{Z}$. Consideremos a la función híbrida:
\begin{equation*}
f:\mathbb{R} \to \mathbb{C}, \quad f(t)=e^{int}.
\end{equation*}Determinemos el valor de la integral $\displaystyle \int_{0}^{2\pi} f(t) \, dt$.

Solución. Es claro que $f$ es una función continua y diferenciable para todo $t\in\mathbb{R}$. Más aún, de acuerdo con la proposición 20.2 tenemos que:
\begin{equation*}
f(t) = e^{int} = \operatorname{cos}(nt) + i \operatorname{sen}(nt),
\end{equation*}por lo que:
\begin{equation*}
f'(t) = in \left[\operatorname{cos}(nt) + i \operatorname{sen}(nt)\right] = ine^{int},
\end{equation*}entonces, la función $F(t)=\dfrac{e^{int}}{in}$ es una primitiva de $f$.

Para $n\neq 0$, por las proposiciones 20.2 y 33.3, tenemos que:
\begin{equation*}
\displaystyle \int_{0}^{2\pi} e^{int} \, dt = \left. \dfrac{e^{int}}{in} \right|_{0}^{2\pi} = \frac{e^{i2n\pi}-e^0}{in} = \frac{1-1}{in} = 0.
\end{equation*}

Mientras que, para $n=0$ tenemos a la función constante $f(t)=1$, entonces, para todo $n\in\mathbb{Z}$ tenemos que:
\begin{equation*}
\displaystyle \int_{0}^{2\pi} e^{int} \, dt= \left\{ \begin{array}{lcc}
0 & \text{si} & n \neq 0, \\ \\
2\pi & \text{si} & n=0.
\end{array} \right.
\end{equation*}

Ejemplo 33.3.
Evaluemos a la integral $\displaystyle \int_{0}^{2\pi} \operatorname{cos}^2(t) \, dt$.

Solución. De acuerdo con la definición 22.1 tenemos que:
\begin{equation*}
\operatorname{cos}^2(t) = \left(\frac{e^{it}+e^{-it}}{2}\right)^2 = \frac{1}{4}\left(e^{i2t} + e^{-i2t} + 2\right).
\end{equation*}

De la proposición 33.1 y el ejemplo anterior se sigue que:
\begin{align*}
\int_{0}^{2\pi} \operatorname{cos}^2(t) \, dt &= \int_{0}^{2\pi} \left[\frac{1}{4}\left(e^{i2t} + e^{-i2t} + 2\right)\right] \, dt\\
& = \frac{1}{4} \left[ \int_{0}^{2\pi} e^{i2t} \, dt + \int_{0}^{2\pi} e^{-i2t} \,dt + \int_{0}^{2\pi} 2 dt\right]\\
& = \frac{1}{4} \left[ 0 + 0 + 4\pi \right]\\
& = \pi.
\end{align*}

Ejemplo 33.4.
Determinemos una primitiva de la función $f(t)$ dada en el ejemplo 33.1. y utilicemos la proposición 33.3 para verificar el resultado del ejemplo 33.1.

Solución. Tenemos que:
\begin{equation*}
f(t)= \left\{ \begin{array}{lcc}
(1+i)t& \text{si} & 0\leq t \leq 1, \\ \\
it^2 & \text{si} & 1\leq t \leq 2.
\end{array} \right.
\end{equation*}

Es claro que la funciones $f_1(t) = (1+i)t$ y $f_2(t) = it^2$ son funciones continuas para todo $t\in\mathbb{R}$, por lo que integrando a cada una de dichas funciones tenemos que:
\begin{equation*}
F(t)= \left\{ \begin{array}{lcc}
\dfrac{(1+i)t^2}{2} + c_1& \text{si} & 0\leq t \leq 1, \\ \\
\dfrac{it^3}{3} + c_2& \text{si} & 1\leq t \leq 2.
\end{array} \right.
\end{equation*}determina una expresión general de las primitivas de $f$, donde $c_1$ y $c_2$ son dos constantes complejas arbitrarias.

Si $c_1 =0$, al evaluar a $F$ en $t=1$ tenemos que:
\begin{equation*}
\dfrac{1+i}{2} =\dfrac{i}{3} + c_2 \quad \Longrightarrow \quad c_2 = \dfrac{1}{2} + \dfrac{i}{6}.
\end{equation*}

Entonces:
\begin{equation*}
F(t)= \left\{ \begin{array}{lcc}
\dfrac{(1+i)t^2}{2} & \text{si} & 0\leq t \leq 1, \\ \\
\dfrac{it^3}{3} + \dfrac{1}{2} + \dfrac{i}{6} & \text{si} & 1\leq t \leq 2.
\end{array} \right.
\end{equation*}es una primitiva de $f$ en el intervalo $[0,2]$.

De acuerdo con la proposición 33.3 tenemos que:
\begin{equation*}
\int_{0}^{2} f(t)\,dt = F(2) – F(0) = \frac{i(2^3)}{3} + \dfrac{1}{2} + \dfrac{i}{6} – 0 = \dfrac{1}{2} + i\dfrac{17}{6},
\end{equation*}lo cual coincide con el resultado del ejemplo 33.1.

Ejemplo 33.5.
Veamos que:
\begin{equation*}
\int_{0}^{1} (t-i)^3 dt = -\frac{5}{4}.
\end{equation*}

Solución. Sea $f(t) = (t-i)^3$. Desarrollando tenemos que:
\begin{equation*}
f(t)=t^3-3t+i(-3t^2+1),
\end{equation*}de donde $u(t)=t^3-3t$ y $v(t)=-3t^2+1$, las cuales son funciones continuas en $[0,1]$, por lo que podemos calcular la integral de cada función. Entonces:
\begin{equation*}
\int_{0}^{1} (t^3-3t) dt = \left.\left(\frac{t^4}{4}-\frac{3t^2}{2}\right)\right|_{0}^{1} = -\frac{5}{4},
\end{equation*}
\begin{equation*}
\int_{0}^{1} (-3t^2+1) dt = \left.\left(-t^3+t\right)\right|_{0}^{1} = 0,
\end{equation*}por lo que:
\begin{equation*}
\int_{0}^{1} (t-i)^3 dt = \int_{0}^{1} (t^3-3t) dt + i \int_{0}^{1} (-3t^2+1) dt = -\frac{5}{4}.
\end{equation*}

Ejemplo 33.6.
Verifiquemos que:
\begin{equation*}
\int_{0}^{\frac{\pi}{2}} e^{t+it} dt = \frac{1}{2}\left[e^{\frac{\pi}{2}}-1+i\left(e^{\frac{\pi}{2}}+1\right)\right].
\end{equation*}

Solución. Sea $f(t) = e^{t+it}$. De acuerdo con la proposición 20.2 tenemos que:
\begin{equation*}
f(t)=e^t\operatorname{cos}(t)+ie^t\operatorname{sen}(t),
\end{equation*}de donde $u(t)=e^t\operatorname{cos}(t)$ y $v(t)=e^t\operatorname{sen}(t)$, las cuales son funciones continuas en $\left[0,\frac{\pi}{2}\right]$, por lo que podemos calcular la integral de cada función. Integrando por partes tenemos que:
\begin{equation*}
\int e^t\operatorname{cos}(t) dt = e^{t}\left[\operatorname{cos}(t) + \operatorname{sen}(t)\right] – \int e^t\operatorname{cos}(t) dt +c,
\end{equation*}de donde:
\begin{equation*}
\int e^t\operatorname{cos}(t) dt = \frac{1}{2} e^{t}\left[\operatorname{cos}(t) + \operatorname{sen}(t)\right] +c.
\end{equation*}

Análogamente tenemos que:
\begin{equation*}
\int e^t\operatorname{sen}(t) dt = \frac{1}{2} e^{t}\left[\operatorname{sen}(t) – \operatorname{cos}(t)\right] +c.
\end{equation*}

Por lo que:
\begin{equation*}
\int_{0}^{\frac{\pi}{2}} e^t\operatorname{cos}(t) dt = \left.\frac{1}{2} e^{t}\left[\operatorname{cos}(t) + \operatorname{sen}(t)\right]\right|_{0}^{\frac{\pi}{2}} = \frac{1}{2}\left(e^{\frac{\pi}{2}}-1\right),
\end{equation*}
\begin{equation*}
\int_{0}^{\frac{\pi}{2}} e^t\operatorname{sen}(t) dt = \left.\frac{1}{2} e^{t}\left[\operatorname{sen}(t) – \operatorname{cos}(t)\right]\right|_{0}^{\frac{\pi}{2}} = \frac{1}{2}\left(e^{\frac{\pi}{2}}+1\right),
\end{equation*}entonces:
\begin{align*}
\int_{0}^{\frac{\pi}{2}} e^{t+it} dt &= \int_{0}^{\frac{\pi}{2}} e^t\operatorname{cos}(t) dt + i \int_{0}^{\frac{\pi}{2}} e^t\operatorname{sen}(t) dt\\
&= \frac{1}{2}\left(e^{\frac{\pi}{2}}-1\right) + i \frac{1}{2}\left(e^{\frac{\pi}{2}}+1\right)\\
& = \frac{1}{2}\left[e^{\frac{\pi}{2}}-1+i\left(e^{\frac{\pi}{2}}+1\right)\right].
\end{align*}

Observación 33.6.
No es difícil verificar que dada una función híbrida continua $f:[a,b]\to\mathbb{C}$, si $F$ y $G$ son dos primitivas de $f$, entonces $F$ y $G$ solo difieren por una constante compleja, en $[a,b]$. Considerando esto y la proposición 33.3, podemos escribir:
\begin{equation*}
\int_{a}^{b} f(t) \, dt = F(t) + c,
\end{equation*}con $c$ una constante compleja, para denotar a cualquier primitiva de $f$.

Ejemplo 33.7.
Sea $z\in\mathbb{C}\setminus\{0\}$. Consideremos a la función híbrida $f(t)=e^{zt}$, con $t\in\mathbb{R}$, entonces:
\begin{equation*}
\int_{a}^{b} e^{zt} \, dt = \frac{1}{z} e^{zt} + c,
\end{equation*}con $c\in\mathbb{C}$ constante, ya que para $F(t) = \dfrac{1}{z} e^{zt}$, por la proposición 32.1(1) y el ejemplo 32.1, se cumple que:
\begin{equation*}
F'(t) = \frac{d}{dt} \dfrac{1}{z} e^{zt} = \dfrac{1}{z} \frac{d}{dt} e^{zt} = \dfrac{1}{z} z e^{zt} = e^{zt}, \quad z\neq 0.
\end{equation*}

Tarea moral

  1. Completa la demostración de la proposición 33.1.
  2. Sean $a,b\in\mathbb{R}\setminus\{0\}$ y sea $f(t)=e^{at}\operatorname{cos}(bt)$. Determina una expresión general para la primitiva de $f$ de las siguientes formas.
    a) Integra por partes dos veces y obtén la solución como en Cálculo.
    b) Expresa $f$ usando la exponencial compleja y utiliza los resultados de esta entrada.
  3. Evalúa las siguientes integrales utilizando los resultados de esta entrada, es decir, sin utilizar integración por partes.
    a) $\displaystyle \int_{0}^{2\pi} e^{3t} \operatorname{cos}^2(2t) dt$.
    b) $\displaystyle \int_{0}^{\pi} e^{t} \operatorname{cos}(3t) \operatorname{sen}(4t)dt$.
  4. Sean $[a,b]\subset{\mathbb{R}}$ un intervalo cerrado, con $a<b$, y $f: [a,b] \to \mathbb{C}$ una función híbrida continua tal que $|f(t)|\leq M$ para todo $t\in[a,b]$, con $M>0$. Prueba que si:
    \begin{equation*}
    \left|\displaystyle \int_{a}^{b} f(t)\, dt\right| = M(b-1),
    \end{equation*}entonces $f(t)=c$, con $c\in\mathbb{C}$ una constante tal que $|c|=M$.
  5. Evalúa las siguientes integrales.
    a) $\displaystyle \int_{0}^{2\pi} e^{i3t} dt$.
    b) $\displaystyle \int_{1}^{2} \operatorname{Log}(it)dt$.
    c) $\displaystyle \int_{-1}^{1} \dfrac{t+i}{t-i} dt$.
    d) $\displaystyle \int_{-1}^{0} \operatorname{sen}(it)dt$.
    e) $\displaystyle \int_{1}^{2} t^{i} dt$, considerando la rama principal de $t^i$.
    f) $\displaystyle \int_{-1}^{1}(2i+3+it)^2 \, dt$.
  6. Sean $m,n\in\mathbb{Z}$. Muestra que:
    \begin{equation*}
    \int_{0}^{2\pi} e^{imt} e^{-int}\, dt = \left\{ \begin{array}{lcc}
    0 & \text{si} & m\neq n, \\ \\
    2 \pi & \text{si} & m=n.
    \end{array} \right.
    \end{equation*}
  7. Evalúa la integral $\displaystyle \int_{-1}^{1} f(t) \, dt$, donde:
    a) $f(t)= \left\{ \begin{array}{lcc}
    (3+2i)t& \text{si} & -1\leq t \leq 0, \\ \\
    it^2 & \text{si} & 0\leq t \leq 1.
    \end{array} \right.$
    b) $f(t)= \left\{ \begin{array}{lcc}
    e^{i\pi t}& \text{si} & -1\leq t \leq 0, \\ \\
    t & \text{si} & 0\leq t \leq 1.
    \end{array} \right.$
  8. Muestra que si $\operatorname{Re}(z)>0$, entonces $\displaystyle \int_{0}^{\infty} e^{-zt} \, dt = \dfrac{1}{z}$.

Más adelante…

En esta entrada hemos definido la integral compleja para una función híbrida y probamos algunas de sus propiedades más importantes que resultan de gran utilidad al resolver ciertos problemas. Es importante mencionar que aunque para el caso de las derivadas y las integrales de funciones híbridas, los resultados parecen ser los mismos que para funciones reales, ya que podemos separar a una función híbrida en su parte real e imaginaria, la aplicación de estos resultados es mucha, en particular para el cálculo de integrales reales a través del uso las propiedades de las funciones complejas como la exponencial y las trigonométricas. Veremos más a detalle estas aplicaciones en la última unidad del curso, aunque muestra de esta utilidad se ve en el ejemplo 33.3.

En la siguiente entrada definiremos lo que es una integral de contorno, que como veremos nos permite hablar de la integrabilidad de una función compleja de variable compleja y aunque dicha definición resulta familiar a la de una integral de línea, veremos que a través de estas integrales obtendremos algunos resultados que serán de suma importancia para la teoría de la variable compleja.

Entradas relacionadas

Variable Compleja I: Trayectorias, curvas y contornos en el plano complejo $\mathbb{C}$

Por Pedro Rivera Herrera

Introducción

A lo largo de la segunda unidad abordamos los conceptos de diferenciabilidad y analicidad para una función compleja. En esta cuarta unidad nuestro objetivo será estudiar el concepto de integración para el caso complejo. Como veremos, muchas de las definiciones y propiedades de las integrales de funciones complejas analíticas estarán sustentadas en muchos resultados de nuestros cursos de Cálculo.

En esta entrada algunas definiciones básicas que nos serán de utilidad a o largo de la unidad para definir la integral de una función compleja.

En la segunda unidad del curso definimos de manera formal el concepto de función compleja de variable compleja y mencionamos que es posible trabajar también con funciones reales de variable compleja o funciones complejas de variable real, en esta unidad éstas últimas tomarán un papel importante.

Definición 32.1. (Funciones híbridas.)
Se llama función híbrida a:

  • una función real de variable compleja, es decir, $f : S\subset\mathbb{C} \to \mathbb{R}$,
  • una función compleja de variable real, es decir, $f : S \subset\mathbb{R} \to \mathbb{C}$.

Observación 32.1.
De forma natural, es claro que para ambos tipos de funciones híbridas hay una noción de continuidad y diferenciabilidad. Sin embargo, solo nos centraremos en definir dichos conceptos para las funciones complejas de variable real, ya que la diferenciabilidad del primer tipo de función híbrida resulta poco interesante, pues una función $f:U \to \mathbb{R}$, con $U\subset\mathbb{C}$ abierto, se puede pensar como una función compleja cuya parte imaginaria es cero y en tal caso, por la proposición 19.3, tendríamos que si $f$ es diferenciable, entonces $f$ debe ser constante.

Por lo tanto, a partir de ahora, al referirnos a una función híbrida asumiremos que estamos considerando a una función compleja de variable real, a menos que se especifique otra cosa.

Observación 32.2.
Dado que $\mathbb{R}\subset\mathbb{C}$, entonces podemos pensar a una función híbrida $f : S \subset\mathbb{R} \to \mathbb{C}$ como una función compleja de variable compleja, cuya parte imaginaria es cero, es decir, $z=t \in \mathbb{R}$. Entonces, por la proposición 12.1, podemos escribir a $f$ como:
\begin{equation*}
f(t) = u(t) + iv(t), \quad \forall t\in S.
\end{equation*}

Definición 32.2. (Continuidad de una función híbrida.)
Sean $(a, b)\subset\mathbb{R}$ un intervalo abierto, $a<b$, y $f : (a, b) \to \mathbb{C}$ una función híbrida. Se dice que $f$ es continua en $t_0 \in (a,b)$ si:
\begin{equation*}
\lim_{t \to t_0} f(t) = f(t_0).
\end{equation*}

Observación 32.3.
De acuerdo con el ejercicio 2 de la entrada 15, tenemos que $f(t) = u(t) + iv(t)$ es una función continua en $(a, b)$ si y solo si las funciones reales $u(t)$ y $v(t)$ son continuas en $(a, b)$.

Definición 32.3. (Continuidad de una función híbrida a trozos.)
Sean $[a, b]\subset\mathbb{R}$ un intervalo cerrado, $a<b$, y $f : [a, b] \to \mathbb{C}$ una función híbrida. Se dice que $f$ es continua a trazos si existe una partición:
\begin{equation*}
P : t_1 < \cdots < t_{n-1},
\end{equation*} del intervalo abierto $(a,b)$ tal que:

  1. $f(t)$ es continua en todo punto en $(a, b)\setminus\{t_1, t_2, \ldots, t_{n-1}\}$ y existen los límites laterales de $f$ en los extremos del intervalo, es decir:
    \begin{equation*}
    \lim_{t \to a^+} f(t) \quad \text{y} \quad \lim_{t \to b^-} f(t).
    \end{equation*}
  2. Existen y son finitos los límites laterales:
    \begin{equation*}
    \lim_{t \to t_k^+} f(t) \quad \text{y} \quad \lim_{t \to t_k^-} f(t),
    \end{equation*}para todo $k=1, 2,\ldots, n-1$.

Definición 32.4. (Diferenciabilidad de una función híbrida.)
Sean $(a, b)\subset\mathbb{R}$ un intervalo abierto, $a<b$, y $f : (a, b) \to \mathbb{C}$ una función híbrida. Se dice que $f$ es diferenciable en $t_0 \in (a,b)$, si es diferenciable en el sentido real, es decir, si existe el límite:
\begin{equation*}
\dfrac{d}{dt} f(t_0) = f'(t_0) = \lim_{t \to t_0} \frac{f(t) – f(t_0)}{t-t_0} = \lim_{h \to 0} \frac{f(t_0 + h) – f(t_0)}{h},
\end{equation*}en tal caso se dice que $f'(t_0)$ es la derivada de $f$ en $t_0$.

Observación 32.4.
Es posible extender el concepto de diferenciabilidad, de una función híbrida $f$, para un intervalo real cerrado $[a, b]\subset\mathbb{R}$ mediante el concepto de límites laterales, es decir, garantizando la existencia de la derivada en $(a,b)$, de acuerdo con la definición 32.4, y de las derivadas laterales de la función $f$, en los puntos extremos del intervalo $[a,b]$, las cuales están dados por:
\begin{equation*}
f'(a) = \lim_{t \to a^{+}} \frac{f(t) – f(a)}{t-a} \quad \text{y} \quad f'(b) = \lim_{t \to b^{-}} \frac{f(t) – f(b)}{t-b}. \tag{32.1}
\end{equation*}

Observación 32.5.
Sea $f(t) = u(t) + iv(t)$, una función híbrida definida en $(a, b)$. De acuerdo con la proposicición 14.2, es claro que para todo $t\in (a, b)$, se tiene que $f'(t)$ existe si y solo si $u'(t)$ y $v'(t)$ existen, y en tal caso:
\begin{equation*}
f'(t) = u'(t) + i v'(t).
\end{equation*}

Las funciones híbridas satisfacen muchas propiedades sobre derivadas similares a las de las funciones reales, las cuales pueden verificarse fácilmente separando a la función híbrida en su parte real e imaginaria.

Proposición 32.1. (Reglas de derivación para funciones híbridas.)
Sean $(a, b)\subset\mathbb{R}$ un intervalo abierto, $a<b$, y $f , g: (a, b) \to \mathbb{C}$ dos funciones híbridas. Entonces para todo $t\in (a,b)$ y todo $\alpha, \beta \in\mathbb{C}$ se cumple que:

  1. $\left(\alpha f + \beta g\right)'(t) = \alpha f'(t) + \beta g'(t)$.
  2. $\left( f \cdot g\right)'(t) = f'(t) g(t) + g'(t) f(t)$.
  3. $\left( \dfrac{f}{g}\right)'(t) = \dfrac{f'(t) g(t) – g'(t) f(t)}{\left(g(t)\right)^2}$, si $g(t)\neq 0$.
  4. Regla de la cadena. Si $h:(c,d) \to (a,b)$ es una función diferenciable, entonces:
    \begin{equation*}
    \left(f \circ h\right)'(t) = f’\left(h(t)\right) h'(t).
    \end{equation*}

Demostración. Se deja como ejercicio al lector.

Ejemplo 32.1.
Sea $z\in\mathbb{C}$. Veamos que para $f(t)=e^{zt}$ se cumple que $f'(t)=ze^{zt}$.

Solución. Sea $z=a+ib$, por la proposición 20.2, tenemos que:
\begin{align*}
f'(t) = \frac{d}{dt} e^{zt} & = \frac{d}{dt} e^{at} e^{ibt}\\
& = \frac{d}{dt} e^{at}\left[\operatorname{cos}(bt)+i\operatorname{sen}(bt)\right]\\
& = \frac{d}{dt} e^{at}\operatorname{cos}(bt)+i\frac{d}{dt}e^{at}\operatorname{sen}(bt)\\
& = ae^{at} \operatorname{cos}(bt) – be^{at} \operatorname{sen}(bt) + i\left[ae^{at} \operatorname{sen}(bt) + be^{at} \operatorname{cos}(bt)\right]\\
& =(a+ib)(e^{at}\operatorname{cos}(bt)+ie^{at}\operatorname{sen}(bt)\\
& = z e^{zt}.
\end{align*}

Por lo que $f$ es una función híbrida diferenciable para todo $t\in\mathbb{R}$.

Observación 32.6.
En la regla de la cadena de la proposición anterior hemos considerado composiciones de funciones de la forma:
\begin{equation*}
\mathbb{R} \to \mathbb{R} \to \mathbb{C}.
\end{equation*}

Sin embargo, es posible establecer una regla de la cadena para composiciones de la forma:
\begin{equation*}
\mathbb{R} \to \mathbb{C} \to \mathbb{C}.
\end{equation*}

Para ello recordemos primeramente el siguiente resultado visto en nuestros cursos de Cálculo.

Teorema 32.1. (Regla de la cadena para funciones reales de varias variables.)
Sean $V \subset \mathbb{R}^n$, $U\subset\mathbb{R}^m$ dos conjuntos abiertos, $\gamma : V \to \mathbb{R}^m$ y $f : U \to \mathbb{R}^k$ dos funciones tales que $\gamma\left(V\right) \subset U$. Si $\gamma$ es diferenciable en $t_0 \in V$ y $f$ es diferenciable en $z_0 = \gamma(t_0) \in U$, entonces $f \circ \gamma$ es diferenciable en $t_0$ y se cumple que:
\begin{equation*}
D \left(f \circ \gamma\right)(t_0) = D f \left(z_0\right) \circ D \gamma\left(t_0\right),
\end{equation*}o equivalentemente:
\begin{equation*}
J \left(f \circ \gamma\right)(t_0) = J f \left(z_0\right) J\gamma\left(t_0\right),
\end{equation*}donde $J f(z_0)$ y $J \gamma(t_0)$ son las matrices Jacobianas, de dimensiones $k \times m$ y $m \times n$, respectivamente.

Considerando el resultado anterior tenemos la siguiente:

Proposición 32.2. (Regla de la cadena para la composición de funciones complejas y funciones híbridas.)
Sean $(a, b)\subset\mathbb{R}$ un intervalo real abierto, $a<b$, y $D\subset\mathbb{C}$ un dominio. Si $\gamma:(a, b) \to D$ es una función diferenciable en $t_0 \in (a, b)$ y $f : D \to \mathbb{C}$ es una función analítica en $z_0 = \gamma(t_0) = x_0 + i y_0 \in D$, entonces la función $\varphi=f\circ \gamma$ es diferenciable en $t_0$ y su derivada es:
\begin{equation*}
\varphi'(t_0) = f'(\gamma(t_0)) \gamma'(t_0). \tag{32.2}
\end{equation*}

Debe ser claro que en la igualdad anterior, el término de la derecha corresponde con el producto de los números complejos $f'(\gamma(t_0))$ y $\gamma'(t_0)$.

Demostración. Dadas las hipótesis, sean $f(z) = u(x, y) + iv(x, y)$ y $\gamma(t) = x(t) + i y(t)$.

Notemos que:
\begin{align*}
\varphi(t) = f(\gamma(t)) & = f(x(t) + i y(t))\\
& = u(x(t), y(t)) + iv(x(t), y(t)), \quad \forall t\in(a, b).
\end{align*}

Como $f$ es analítica en $z_0 \in D$, por el teorema 18.3, tenemos que $f$ es dierenciable en el sentido real en $z_0$ y se satisfacen las ecuaciones de C-R en dicho punto. Entonces, de la regla de la cadena para funciones reales de varias variables se sigue que $\varphi = f\circ \gamma$ es diferenciable en $z_0$ y se cumple que:
\begin{equation*}
\varphi'(t_0) = J \left(f \circ \gamma\right)(t_0) = J f \left(z_0\right) J\gamma\left(t_0\right).
\end{equation*}

Dado que se cumplen las ecuaciones de C-R en $z_0 \in D$, entonces, por el ejercicio 3 de la entrada 12, tenemos que:
\begin{align*}
J f \left(z_0\right) & = \begin{pmatrix}
u_x(z_0) & u_y(z_0)\\
v_x(z_0) & v_y(z_0)
\end{pmatrix}\\
& = \begin{pmatrix}
u_x(\gamma(t_0)) & -v_x(\gamma(t_0))\\
v_x(\gamma(t_0)) & u_x(\gamma(t_0))
\end{pmatrix}\\
& = u_x(\gamma(t_0)) + i v_x(\gamma(t_0))\\
& = f'(\gamma(t_0)),
\end{align*}\begin{equation*}
J \gamma\left(t_0\right) = \begin{pmatrix}
x'(t_0)\\
y'(t_0)
\end{pmatrix} = x'(t_0) + i y'(t_0) = \gamma'(t_0),
\end{equation*}por lo que:
\begin{equation*}
\varphi'(t_0) = f'(\gamma(t_0)) \gamma'(t_0).
\end{equation*}

$\blacksquare$

A partir de ahora, nos centraremos en las funciones híbridas que son continuas, ya que como veremos nos serán de gran utilidad al definir el concepto de integral desde la perspectiva compleja. Para hacer clara la notación, utilizaremos letras griegas para hablar de funciones híbridas y reservaremos las letras usuales $f$ y $g$ para hablar de funciones complejas de variable compleja.

Definición 32.5. (Trayectoria y curva en $\mathbb{C}$.)
Sea $[a, b]\subset\mathbb{R}$, con $a<b$, un intervalo real cerrado. A toda función continua $\gamma: [a,b] \to \mathbb{C}$ se le llama una trayectoria en el plano complejo $\mathbb{C}$. A la imagen de $[a, b]$ bajo $\gamma$, es decir, el conjunto:
\begin{equation*}
\Gamma := \gamma\left([a,b]\right) = \left \{\gamma(t) : t\in [a,b] \right\} \subset \mathbb{C},
\end{equation*} se le llama la imagen de la trayectoria o la curva descrita por $\gamma$.

Observación 32.7.
Si consideramos a $z=x+iy \in\mathbb{C}$, tiene sentido adoptar la notación $z(t) = x(t) + iy(t)$ para referirnos a una trayectoria, cuya curva está determinada por cada valor de $t\in [a,b]$, la cual decimos que está parametrizada por las funciones reales continuas $x(t)$ y $y(t)$, definidas en $[a,b]$, a las que llamaremos funciones paramétricas o funciones componentes de $z(t)$. Así, una curva $\gamma$ en $\mathbb{C}$ puede parametrizarse mediante sus funciones componentes como:
\begin{align*}
\gamma(t) &= (x(t), y(t))\\
&= x(t) + iy(t) \in \mathbb{C}, \quad \forall t\in [a,b]. \tag{32.3}
\end{align*}

Observación 32.8.
El uso de la notación $\Gamma$ para referirnos a la curva asociada a la trayectoria $\gamma$ se usará únicamente cuando sea necesario especificar al conjunto de manera explícita. En general, cuando hablemos de $\gamma$ se sobrentenderá que nos referimos a la trayectoria (función híbrida continua) o a su curva asociada (imagen de $\gamma$), por lo que usaremos de forma indistinta la notación y aunque escribamos a $\gamma$ como en (32.3) no debemos confundirnos entre una curva $\Gamma$ y su parametrización $\gamma$. Es común usar alguna letra griega para denotar a alguna curva.

Definición 32.6. (Curva cerrada, orientación, curva simple y curva cerrada simple.)
Sea $\gamma: [a,b] \to \mathbb{C}$, con $a<b$, una trayectoria dada por $\gamma(t) = x(t) + iy(t) $, para $t\in [a, b]$. Al punto $\gamma(a) = (x(a), y(a))$ se le llama el {\bf origen} de la curva $\gamma$, mientras que al punto $\gamma(b) = (x(b), y(b))$ se le llama el extremo o final de $\gamma$. Para denotar este sentido en la curva, de manera gráfica se suelen utilizar algunas flechas sobre la gráfica, figura 110, indicando que la curva inicia en $\gamma(a)$ y termina en $\gamma(b)$. Si el origen y el extremo de $\gamma$ son el mismo punto, es decir $\gamma(a) = \gamma(b)$, se dice que la curva es cerrada.

Por otra parte, se dice que el sentido de una curva cerrada o de algún arco de curva tiene orientación o dirección positiva si se recorre en el sentido contrario al de las manecillas del reloj, figura 111, y en caso contrario se dice que tiene orientación o dirección negativa.

Por último, a una curva $\gamma$ que no se corte asimisma, excepto quizás en sus extremos, es decir, $\gamma(t_1) \neq \gamma(t_2)$ si $t_1,t_2\in(a,b)$ y $t_1\neq t_2$, se le llama una curva simple. Si la curva $\gamma$ es cerrada y no se corte asimisma, excepto en sus extremos, se le llama una curva cerrada simple o una curva de Jordan.

Figura 110: Gráfica de la imagen de una curva $\gamma$ definida sobre el intervalo $[a, b]$ y con valores en el plano complejo $\mathbb{C}$.
Figura 111: Gráfica de la orientación positiva y negativa de una curva cerrada en el plano complejo $\mathbb{C}$.

Observación 32.9.
Debe ser claro que una curva es simple o cerrada simple, si la función $\gamma$ que la describe es una función inyectiva en $[a,b]$, es decir, su curva asociada no se corta asimisma en más de un punto, correspondiente con sus extremos $\gamma(a) = \gamma(b)$, como podemos ver en el ejemplo 32.3. Aunque no necesariamente una trayectoria debe ser una función inyectiva, por lo que puede suceder que su curva asociada se corte asimisma en más de un punto, incluyendo los extremos, es decir, que la curva se cerrada, pero no simple, como podemos ver en el ejemplo 32.4.

Definición 32.7. (Curva de clase $C^1$ o suave.)
Sea $\gamma : [a, b] \to \mathbb{C}$, con $a<b$, una trayectoria. Se dice que la curva asociada a $\gamma$ es continuamente diferenciable, de clase $C^1$ o suave, lo cual se denota como $\gamma \in C^1\left([a, b]\right)$, si:

  1. $\gamma$ es diferenciable en $[a, b]$, de acuerdo con la observación 32.4.
  2. $\gamma’$ es continua en $[a, b]$, es decir, $\gamma’$ es continua en $(a, b)$, y existen los límites laterales de $\gamma’$ en los extremos del intervalo, es decir:
    \begin{equation*}
    \lim_{t \to a^+} \gamma'(t) \quad \text{y} \quad \lim_{t \to b^-} \gamma'(t).
    \end{equation*}

Observación 32.10.
Es importante mencionar que en algunos textos, el término «suave» suele utilizarse para las curvas tales que $\gamma'(t)$ existe y $\gamma'(t) \neq 0$ para todo $t\in[a,b]$, definición 32.8. Sin embargo, para los fines del curso, dicho término se utilizará para hablar de una trayectoria $\gamma(t)$ de clase $C^1([a,b])$.

Definición 32.8. (Curva regular.)
Sea $\gamma: [a,b] \to \mathbb{C}$ una trayectoria, con $a<b$. Se dice que $\gamma$ es una curva regular si:

  1. $\gamma'(t)$ existe y es continua en $[a,b]$.
  2. $\gamma'(t) \neq 0$ para todo $t\in(a,b)$.

Observación 32.11.
Para una curva $\gamma$, definida en un intervalo $[a,b]$, tal que $\gamma'(t) \neq 0$ para algún $t\in[a,b]$, debe ser claro que su derivada tiene una interpretación geométrica correspondiente con el vector tangente a la curva $\gamma$ en el punto $\gamma(t)$.

Figura 112: Gráfica de dos vectores tangentes a una curva simple $\gamma$ en los puntos $\gamma(t_0)$ y $\gamma(t_1)$, para $t_0, t_1 \in [a,b]$ tales que $\gamma'(t_0)\neq 0$ y $\gamma(t_1)\neq 0$.

Analicemos los siguientes ejemplos de curvas en el plano complejo $\mathbb{C}$ con las que ya estamos familiarizados.

Ejemplo 32.2. (Segmentos de recta.)
Sean $z_1,z_2\in\mathbb{C}$ con $z_1\neq z_2$. Definimos al segmento de recta con origen $z_1$ y final $z_2$ como la trayectoria:
\begin{equation*}
[z_1, z_2] : [0,1] \to \mathbb{C}, \quad [z_1, z_2](t) := z_1 + (z_2-z_1)t, \quad \forall t\in[0,1].
\end{equation*}

Si $z_1 = a_1 + ib_1$ y $z_2 = a_2 + ib_2$, entonces una parametrización de dicho segmento es $[z_1, z_2](t) = x(t) + iy(t)$, donde:
\begin{equation*}
x(t):=(a_2-a_1)t + a_1, \quad y(t):=(b_2-b_1)t + b_1, \quad t\in[0,1].
\end{equation*}

Notemos que dicha curva es regular ya que $x'(t) = a_2 – a_1 \neq 0$ y $y'(t) = b_2 – b_1 \neq 0$ para todo $ t\in[0,1]$.

Es claro que hemos hecho un abuso en la notación, por lo que es importante no confundir la notación utilizada para la trayectoria $[z,w]$ con la de un intervalo real cerrado.

Observación 32.12.
Es importante enfatizar en el hecho de que una trayectoria puede tener distintas parametrizaciones, pero la misma curva asociada. Por lo que, siempre que sea necesario, se debe especificar la parametrización de la curva con la que se está trabajando.

Ejemplo 32.3.
Consideremos al segmento $[0,1]$. Es claro que la imagen de dicha curva es un segmento de recta en el plano. Sin embargo, notemos que dicha curva puede parametrizarse como:
\begin{align*}
\gamma_1(t) = t, \quad 0\leq t \leq 1,\\
\gamma_2(t) = t^2, \quad 0\leq t \leq 1,\\
\gamma_3(t) = \frac{t}{2}, \quad 0\leq t \leq 2.
\end{align*}

Ejemplo 32.4. (Circunferencias.)
Un ejemplo sencillo de una curva cerrada simple es la circunferencia unitaria $C(0,1)$, con orientación positiva, figura 113, cuya parametrización está dada por:
\begin{equation*}
\gamma(t) = e^{it} = \operatorname{cos}(t) + i \operatorname{sen}(t), \quad 0\leq t \leq 2\pi.
\end{equation*}

Esta curva cerrada es simple y tiene como punto inicial a $\gamma(0) =1$ y punto final a $\gamma(2\pi) =1$.

Figura 113: Gráfica de la circunferencia unitaria orientada positivamente.

En general, una circunferencia $C(z_0,r)$, con $r>0$ y $z_0\in\mathbb{C}$ fijo, es un ejemplo de una curva cerrada simple en el plano complejo.

Sean $z_0=a+ib\in\mathbb{C}$ fijo y $r>0$. El conjunto:
\begin{equation*}
C(z_0,r) = \left\{ z = a+r\operatorname{cos}(t) + \left[ b+r\operatorname{sen}(t) \right] \in \mathbb{C} : t\in[0,2\pi]\right\},
\end{equation*}corresponde con la imagen de la trayectoria $ \gamma(t) = z_0 + re^{it}$, con $t\in[0, 2\pi]$, que describe una circunferencia de radio $r>0$ y centro $z_0 = a+ib$, la cual es recorrida en sentido positivo, figura 114, y tiene como parametrización:
\begin{align*}
\gamma(t) & = z_0 + re^{it}\\
& = a +ib + r\left[\operatorname{cos}(t) + i\operatorname{sen}(t)\right]\\
& = a + r\operatorname{cos}(t) + i \left[b +\operatorname{sen}(t)\right], \quad t\in[0, 2\pi],
\end{align*}es decir, $\gamma(t) = x(t) + iy(t)$, donde:
\begin{equation*}
x(t) = a + r\operatorname{cos}(t), \quad y(t) = b +\operatorname{sen}(t), \quad t\in[0, 2\pi].
\end{equation*}

Esta curva cerrada tiene como punto inicial y final a:
\begin{equation*}
z_0 + r = \gamma(0) = \gamma(2\pi).
\end{equation*}

No es difícil verificar que una circunferencia también es una curva regular, por lo que se deja como ejercicio al lector.

Figura 114: Gráfica de una circunferencia de radio $r>0$ y centro $z_0$, orientada positivamente.

Ejemplo 32.5.
Consideremos a la lemniscata, la cual es una curva en el plano $\mathbb{R}^2$ descrita por la ecuación polar:
\begin{equation*}
r^2 = 2d^2 \operatorname{cos}(2\theta),
\end{equation*}donde $d>0$ es una constante.

Dado que para $\theta \in \left(\dfrac{\pi}{4}, \dfrac{3 \pi}{4}\right) \bigcup \left(-\dfrac{3\pi}{4}, -\dfrac{\pi}{4}\right)$ se tiene que $\operatorname{cos}(2\theta)<0$, entonces para dichos valores de $\theta$ la curva no tiene puntos en el plano.

Una posible parametrización para dicha curva, en el plano complejo $\mathbb{C}$, es $\gamma(t) = x(t)+iy(t)$, donde:
\begin{equation*}
x(t) := \frac{d\sqrt{2}\operatorname{cos}(t)}{1+\operatorname{sen}^2(t)}, \quad y(t) := \frac{d\sqrt{2}\operatorname{sen}(t)\operatorname{cos}(t)}{1+\operatorname{sen}^2(t)}, \quad t\in[0,2\pi].
\end{equation*}

Bajo esta parametrización la curva $\gamma$ tiene una orientación positiva, figura 115. Más aún, como:
\begin{equation*}
\gamma(0) = d\sqrt{2} =\gamma(2\pi),
\end{equation*}entonces dicha curva es cerrada. Sin embargo, la lemniscata no es una curva simple ya que:
\begin{equation*}
\gamma\left(\dfrac{\pi}{2}\right) = 0 = \gamma\left(\dfrac{3\pi}{2}\right),
\end{equation*}con $\dfrac{\pi}{2}, \dfrac{3\pi}{2} \in [0, 2\pi]$ y $\dfrac{\pi}{2} \neq \dfrac{3\pi}{2}$, es decir, $\gamma$ no es inyectiva.

Figura 115: Gráfica de una lemniscata, con $d>0$ constante, la cual es una curva cerrada, pero no simple.

Definición 32.9.
Sea $\gamma : [a,b] \to \mathbb{C}$ una trayectoria y $D \subset \mathbb{C}$ un dominio. Si la curva descrita por $\gamma$ está contenida en $D$, es decir, si la imagen de $\gamma$ es tal que:
\begin{equation*}
\{ \gamma(t) : t\in [a, b]\} \subset D,
\end{equation*}se dice que $\gamma$ es una curva en $D$, lo cual se denota como $\gamma : [a,b] \to D$.

Ejemplo 32.6.
El circulo unitario $\gamma(t) = \operatorname{cos}(t) + i\operatorname{sen}(t)$, con $0 \leq t \leq 2\pi$, figura 116, es una curva en el dominio:
\begin{equation*}
D = \left\{z \in \mathbb{C} : \frac{1}{2} \leq |z| \leq 2 \right\}.
\end{equation*}

Figura 116: Circunferencia unitaria en el dominio $D$.

Como mencionamos antes, la continuidad de las funciones híbridas resulta fundamental para definir los conceptos básicos para la teoría de la integral compleja.

Definición 32.10 (Curva de clase $C^1$ o suave a trozos.)
Una trayectoria $\gamma: [a,b] \to \mathbb{C}$ es llamada continuamente diferenciable por partes o a trozos, de clase $C^1$ por partes o a trozos o suave por partes o a trozos si existe una partición:
\begin{equation*}
P : a=t_0 < t_1 < \cdots < t_{n-1}<t_n=b,
\end{equation*}del intervalo $[a,b]$ tal que la restricción de $\gamma$ a cada subintervalo $[t_{k-1}, t_k]$, con $1\leq k \leq n$, es de clase $C^1$ o suave, definición 32.7.

En otras palabras, $\gamma$ es continua en $[a,b]$ y para $1\leq k \leq n$:

  1. $\gamma'(t)$ existe para todo $t\in(t_{k-1}, t_k)$ y en los puntos extremos $t_{k-1}$ y $t_k$ existen los límites laterales (32.1).
  2. $\gamma'(t)$ es continua en cada intervalo $[t_{k-1}, t_k]$.

Observación 32.13.
Debe ser claro que si $\gamma’$ está definida en $[a,b]$, entonces $\gamma’$ puede tener discontinuidades de salto en algunos $t_k$, ya que aunque existan los límites laterales:
\begin{equation*}
\lim_{t \to t_k^+} \gamma'(t) \quad \text{y} \quad \lim_{t \to t_k^-} \gamma'(t),
\end{equation*}estos pueden no ser iguales.

Definición 32.11. (Contorno o camino en $\mathbb{C}$.)
Se define a un contorno o camino en el plano complejo $\mathbb{C}$, como la unión finita de curvas de clase $C^1$ o de clase $C^1$ a trozos.Es decir, un contorno es una sucesión de curvas suaves $\left\{\gamma_1, \ldots, \gamma_n\right\}$ tal que el punto final de $\gamma_k$ coincide con el punto de origen de $\gamma_{k+1}$, para $1\leq k \leq n-1$.

Es común denotar a un contorno a través de la letra $C$, aunque igual puede utilizarse alguna letra griega.

Ejemplo 32.7.
Un ejemplo sencillo de un camino en $\mathbb{C}$ es una poligonal, definición 10.5, la cual podemos pensar como una curva suave a trozos dada por la unión finita de los segmentos de recta $[z_1, z_2], [z_2, z_3], \ldots, [z_{n-1}, z_n]$, cuyo punto de origen es $z_1$ y su punto final es $z_n$. Al igual que cualquier curva, una poligonal puede ser simple, cerrada o cerrada simple.

Si consideramos a $z_1=0, z_2 = 1+i$ y $z_3=-1+i$, entonces la poligonal $[z_1, z_2, z_3, z_1]$ resulta ser un camino cerrado simple, figura 117.

Figura 117: Gráfica de la poligonal $[z_1, z_2, z_3, z_1]$.

Podemos describir al camino $\gamma$, dado por la poligonal $[z_1, z_2, z_3, z_1]$, parametrizando a los segmentos $[z_1, z_2]$, $[z_2, z_3]$ y $[z_3, z_1]$, como las curvas suaves $\gamma_1$, $\gamma_2$ y $\gamma_3$, definidas en el intervalo $[0,1]$, dadas por:
\begin{align*}
\gamma_1(t):=[z_1, z_2](t) & = 0+(1+i-0)t = (1+i)t,\\
\gamma_2(t):=[z_2, z_3](t) & = 1+i+[-1+i-(1+i)]t = 1+i-2t,\\
\gamma_3(t):=[z_3, z_1](t) & = -1+i+[0-(-1+i)]t = (-1+i)(1-t).
\end{align*}

Ejemplo 32.8.
Veamos que la curva dada por $\gamma(t)=t+i|t|$, con $t\in[-1,1]$, no es una curva suave, sin embargo sí es un camino, es decir, una curva suave a trozos.

Solución. Primeramente tenemos que los puntos de origen y final de la curva son, respectivamente:
\begin{equation*}
\gamma(-1)=-1+i|-1|=-1+i \quad \text{y} \quad \gamma(1)=1+i|1|=1+i.
\end{equation*}

Es claro que $\gamma(t)=t+i|t|$ es una trayectoria ya que sus funciones paramétricas $x(t)=t$ y $y(t)=|t|$ son continuas en $[-1,1]$. No es difícil verificar que $\gamma$ es inyectiva y por tanto simple, por lo que se deja como ejercicio al lector.

Podemos visualizar la curva asociada a $\gamma$ en la figura 118.

Figura 118: Gráfica de la curva $\gamma(t)=t+i|t|$, con $t\in[-1,1]$.

Gráficamente es claro que en $t=0$ la trayectoria $\gamma$ no es diferenciable. Procedemos a verificarlo de forma analítica. Dado que:
\begin{align*}
\lim_{t\to 0^-} \frac{y(t)-y(0)}{t-0} &= \lim_{t\to 0^-} \frac{|t|}{t} = \lim_{t\to 0^-} \frac{-t}{t} = -1,\\
\lim_{t\to 0^+} \frac{y(t)-y(0)}{t-0} &= \lim_{t\to 0^+} \frac{|t|}{t} = \lim_{t\to 0^+} \frac{t}{t} = 1,
\end{align*}tenemos que $y'(0)$ no existe, entonces, por la observación 32.5, $\gamma'(0)$ no existe, por lo que $\gamma$ no es una curva suave en $[-1,1]$. Sin embargo, notemos que podemos ver a la trayectoria $\gamma$ como:
\begin{equation*}
\gamma(t)= \left\{ \begin{array}{lcc} t-it & \text{si} & -1 \leq t \leq 0 \\ \\ t+it & \text{si} & 0 \leq t \leq 1 \end{array} \right.
\end{equation*}

Veamos entonces que bajo la restricción de $\gamma$ a los intervalos $[-1,0]$ y $[0,1]$ obtenemos curvas suaves.

Para $t\in[-1,0]$ tenemos que $\gamma(t)=t-it$ es una función continua. Más aún, para $t_0\in(-1,0)$ tenemos que:
\begin{equation*}
\gamma'(t_0) = \lim_{t\to t_0} \frac{\gamma(t)-\gamma(t_0)}{t-t_0} = \lim_{t\to t_0} \frac{(t-t_0)(1-i)}{t-t_0} = 1-i.
\end{equation*}

Mientras que:
\begin{align*}
\gamma'(-1) &= \lim_{t \to -1^{+}} \frac{\gamma(t)-\gamma(-1)}{t-(-1)} = \lim_{t \to -1^{+}} \frac{(1-i)(t+1)}{t+1} = 1-i,\\
\gamma'(0) &= \lim_{t \to 0^{-}} \frac{\gamma(t)-\gamma(0)}{t-0} = \lim_{t \to 0^{-}} \frac{t-it}{t} = 1-i,
\end{align*}por lo que es claro que $\gamma'(t)$ existe para todo $t\in[-1,0]$ y es una función continua en dicho intervalo, entonces para $t\in[-1,0]$ tenemos que $\gamma(t)=t-it$ es una curva suave.

De manera análoga, se puede verificar que para $t\in[0,1]$ la trayectoria $\gamma(t)=t+it$ es una curva suave. Por lo tanto, la curva dada por $\gamma(t)=t+i|t|$, con $t\in[-1,1]$, es una curva suave a trazos, es decir, es un camino en $\mathbb{C}$.

Ejemplo 32.9.
El contorno $C$ dado en la figura 119 está formado por las curvas suaves $\gamma_1(t)=t, \gamma_2(t)=e^{i\frac{\pi}{2}(1-t)}$ y $\gamma_3(t)=t$ determinadas por el segmento de recta $[-2, -1]$, la semicircunferencia unitaria en el semiplano superior y el segmento de recta $[1, 2]$, respectivamente.

Podemos parametrizar al contorno $C$ mediante la trayectoria:
\begin{equation*}
\gamma:[-2,2] \to \mathbb{C},
\end{equation*}dada por:
\begin{equation*}
\gamma(t) = \left\{ \begin{array}{lcc}
t & \text{si} & -2 \leq t \leq -1,\\
\\ e^{i\frac{\pi}{2}(1-t)}, & \text{si} & -1 \leq t \leq 1, \\
\\ t, & \text{si} & 1 \leq t \leq 2. \end{array} \right.
\end{equation*}

Figura 119: Gráfica del contorno $C$ dado por la trayectoria $\gamma$ del ejemplo 32.8.

Existen dos métodos elementales para modificar o combinar curvas, con el objetivo de obtener nuevas curvas.

Definición 32.12. (Curva opuesta.)
Si $\gamma: [a,b] \to \mathbb{C}$ es una trayectoria, con $a<b$, se define a la curva opuesta de $\gamma$, la cual se denota como $-\gamma$, a la trayectoria:
\begin{equation*}
-\gamma:[a,b]: \to \mathbb{C},
\end{equation*}dada por:
\begin{equation*}
-\gamma(t) := \gamma(b+a-t), \quad \forall t\in[a,b]. \tag{32.4}
\end{equation*}

Debe ser claro que la curva asociada a $\gamma$ y a $-\gamma$ es la misma, pero $\gamma(t)$ y $-\gamma(t)$ recorren dicha curva en sentido contrario, es decir, la orientación de una respecto a la otra es opuesta conforme $t$ toma valores en $[a,b]$. En particular, el punto de origen de una es el punto final de la otra.

Observación 32.14.
Se puede probar que si $\gamma$ es una curva suave a trozos, entonces $-\gamma$ también es una curva suave a trozos, ejercicio 3.

Ejemplo 32.10.
Determinemos la curva opuesta del segmento de recta $[z_1, z_2]$, con $z_1 \neq z_2$.

Solución. De acuerdo con el ejemplo 32.2 sabemos que una parametrización del segmento $[z_1, z_2]$ está dada por la trayectoria:
\begin{equation*}
\gamma:[0,1]: \to \mathbb{C},
\end{equation*}dada por:
\begin{equation*}
\gamma(t) = z_1 + (z_2-z_1)t, \quad \forall t\in[0,1],
\end{equation*}por lo que, considerando (32.4) tenemos que:
\begin{equation*}
-\gamma(t) = \gamma(1-t) = z_1 + (z_2-z_1)(1-t) = z_2 + (z_1-z_2)t, \quad \forall t\in[0,1].
\end{equation*}

Definición 32.13. (Curva suma o yuxtaposición.)
Si $\gamma_1: [a_1,b_1] \to \mathbb{C}$ y $\gamma_2: [a_2,b_2] \to \mathbb{C}$, con $a_i<b_i$ para $i=1, 2$, son dos trayectorias tales que $\gamma_1(b_1) = \gamma_2(a_2)$, se define a la curva suma o a la yuxtaposición de $\gamma_1$ y $\gamma_2$, la cual se denota como $\gamma_1+\gamma_2$, a la trayectoria:
\begin{equation*}
\gamma_1+\gamma_2:[a_1,b_1+b_2-a_2]: \to \mathbb{C},
\end{equation*}dada por:
\begin{equation*}
(\gamma_1+\gamma_2)(t) :=\left\{ \begin{array}{lcc}
\gamma_1(t), & \text{si} & a_1 \leq t \leq b_1,\\ \\ \gamma_2(t-b_1+a_2), & \text{si} & b_1 \leq t \leq b_1 +b_2 – a_2. \end{array} \right.
\end{equation*}

La curva asociada a la trayectoria $\gamma_1+\gamma_2$ corresponde con la curva de $\gamma_1$ seguida de la de $\gamma_2$.

Observación 32.15.
En general, se puede definir por inducción la suma finita $\gamma_1 + \cdots + \gamma_n$, de las trayectorias $\gamma_k: [a_k,b_k] \to \mathbb{C}$, con $1\leq k \leq n$, tales que $\gamma_j(b_j) = \gamma_{j+1}(a_{j+1})$ para $j=1,2,\ldots,n-1$. De hecho se puede probar que la suma de trayectorias de clase $C^1$ a trozos es también una trayectoria suave, ejercicio 3. Más aún, se puede verificar que una trayectoria $\gamma$ de clase $C^1$ a trozos se puede expresar como la suma finita de trayectorias suaves, es decir, $\gamma = \gamma_1 + \cdots + \gamma_n$, ejercicio 6.

Ejemplo 32.11.
De acuerdo con la definición 32.13 y la observación 32.15, no es difícil verificar que el camino $\gamma$ del ejemplo 32.7, que describe a la poligonal $[z_1, z_2, z_3, z_1]$, con $z_1=0, z_2 = 1+i$ y $z_3=-1+i$, figura 117, se puede ver como la suma de las curvas suaves:
\begin{align*}
\gamma_1(t) = (1+i)t, \quad \gamma_2(t) = 1+i-2t, \quad \gamma_3(t) = (-1+i)(1-t), \quad \forall t\in[0,1],
\end{align*}es decir:
\begin{equation*}
\gamma := (\gamma_1+\gamma_2 +\gamma_3) : [0, 3] \to \mathbb{C}
\end{equation*}dada por:
\begin{equation*}
\gamma(t) = (\gamma_1+\gamma_2+\gamma_3)(t) =\left\{ \begin{array}{lcc}
(1+i)t & \text{si} & 0 \leq t \leq 1,\\
\\ 3+i-2t, & \text{si} & 1 \leq t \leq 2, \\
\\ (-1+i)(3-t), & \text{si} & 2 \leq t \leq 3. \end{array} \right.
\end{equation*}

Ejemplo 32.12.
Consideremos a la curva $\gamma:[0,4]\to\mathbb{C}$ dada por:
\begin{equation*}
\gamma(t) = \left\{ \begin{array}{lcc}
t & \text{si} & 0 \leq t \leq 1,\\
\\ 1+i(t-1), & \text{si} & 1 \leq t \leq 2, \\
\\ 3-t+i, & \text{si} & 2 \leq t \leq 3, \\
\\ i(4-t), & \text{si} & 3 \leq t \leq 4. \end{array} \right.
\end{equation*}

Dicha curva es un contorno cerrado simple, ya que es una curva suave a trozos, cerrada simple. Más aún, dicha curva corresponde con la frontera del cuadrado con vértices en $z_1=0, z_2=1, z_3=1+i$ y $z_4=i$, la cual puede verse como la suma de los segmentos de recta $[z_1, z_2]$, $[z_2, z_3]$ y $[z_3, z_1]$, es decir, como la poligonal cerrada $[z_1, z_2, z_3, z_1]$, figura 120.

Figura 120: Gráfica del contorno $C$ correspondiente con la frontera del cuadrado con vértices en $z_1=0, z_2=1, z_3=1+i$ y $z_4=i$.

Definición 32.14. (Reparametrización de una curva en $\mathbb{C}$.)
Sean $\gamma: [a,b] \to \mathbb{C}$ y $\beta: [c,d] \to \mathbb{C}$, con $a<b$ y $c<d$, dos trayectorias suaves. Si existe una biyección $\sigma:[c,d] \to [a,b]$ continua y de clase $C^1$, tal que $\sigma$ es creciente y $\beta=\gamma\circ \sigma$, se dice que $\beta$ es una reparametrización de $\gamma$.

Observación 32.16.
De la definición anterior debe ser claro que aunque la parametrización de $\gamma$ y $\beta$ sea distinta, ambas trayectorias tienen la misma curva asociada y existe una relación entre la parametrización de cada trayectoria descrita por la función $\sigma$. Más aún, se puede verificar que si $\sigma$ es una función de clase $C^1$ o de clase $C^1$ a trozos, se tiene una reparametrización de clase $C^1$ o de clase $C^1$ a trozos, respectivamente.

Ejemplo 32.13.
Veamos que las siguientes trayectorias $\beta$ son una reparametrización de la trayectorias $\gamma$ dadas.

a) Sean $z_1, z_2\in\mathbb{C}$ tales que $z_1 \neq z_2$. Sean $\beta:\left[0, k\right] \to \mathbb{C}$, con $k=|z_2-z_1|>0$, y $\gamma:[0,1] \to \mathbb{C}$ dadas, respectivamente, por:
\begin{equation*}
\beta(t) = z_1 + \frac{(z_2-z_1)t}{k}, \quad \gamma(t) = z_1 + (z_2-z_1)t.
\end{equation*}

Solución. Sea $\sigma:[0,k]\to[0,1]$ dada por $\sigma(t)=\dfrac{t}{k}$, con $k=|z_2-z_1|>0$. Claramente $\sigma$ es una función biyectiva continua, de clase $C^1$ con derivada $\sigma'(t)=\dfrac{1}{k}>0$ para todo $t\in[0,k]$, por lo que $\sigma$ es una función creciente. Más aún, tenemos que:
\begin{equation*}
\gamma(\sigma(t))= \gamma\left(\dfrac{t}{k}\right) = z_1 + \frac{(z_2-z_1)t}{k} = \beta(t), \quad \forall t\in[0,k],
\end{equation*}por lo que $\beta$ es una reparametrización de $\gamma$.

b) Sean $z_0\in\mathbb{C}$ fijo y $r>0$. Sean $\gamma:\left[0, 2\pi\right] \to \mathbb{C}$ y $\beta:[0,1] \to \mathbb{C}$ dadas, respectivamente, por:
\begin{equation*}
\gamma(t) = z_0 + re^{it}, \quad \beta(t) = z_0 + re^{i2\pi t}.
\end{equation*}

Solución. Sea $\sigma:[0,1]\to[0,2\pi]$ dada por $\sigma(t)=2\pi t$. Es claro que $\sigma$ es una función biyectiva continua, de clase $C^1$ con derivada $\sigma'(t)=2\pi>0$ para todo $t\in[0,1]$, por lo que $\sigma$ es una función creciente. Tenemos que:
\begin{equation*}
\gamma(\sigma(t))= \gamma\left(2\pi t\right) = z_0 + re^{i2\pi t} = \beta(t), \quad \forall t\in[0,1],
\end{equation*}por lo que $\beta$ es una reparametrización de $\gamma$.

De nuestros cursos de Cálculo sabemos que la longitud del arco de una curva es una cantidad bastante útil. Recordemos que si $\gamma:[a,b] \to\mathbb{R}^2$ es una curva suave parametrizada por las funciones reales $x, y:[a,b] \to\mathbb{R}$, es decir, $\gamma(t) = (x(t), y(t))$, entonces la longitud del arco de dicha curva se puede obtener como:
\begin{equation*}
\int_{a}^{b} \sqrt{[x'(t)]^2+[y'(t)]^2} \,dt. \tag{32.5}
\end{equation*}

Motivados en lo anterior tenemos la siguiente:

Definición 32.15. (Longitud de una curva en $\mathbb{C}$.)
Sea $\gamma: [a,b] \to \mathbb{C}$, con $a<b$, una trayectoria suave a trozos. Se define la longitud de $\gamma$ como:
\begin{equation*}
\ell(\gamma):= \int_{a}^{b}|\gamma'(t)| \, dt.
\end{equation*}

Definición 32.16. (Curva rectificable en $\mathbb{C}$.)
Sea $\gamma: [a,b] \to \mathbb{C}$, con $a<b$, una trayectoria suave a trozos. Se dice que la curva $\gamma$ es rectificable si tiene longitud finita.

Observación 32.17.
Geométricamente es claro que $\ell(\gamma)$ es igual a longitud del arco de la curva asociada a $\gamma$.

Sabemos que la longitud de la curva $\gamma(t)=x(t)+iy(t)$ se puede aproximar sumando la longitud de segmentos de rectas dados por puntos sobre la curva, es decir, obteniendo la longitud de una poligonal formada por una partición:
\begin{equation*}
P : a=t_0 < t_1 < \cdots < t_{n-1}<t_n=b,
\end{equation*}del intervalo $[a,b]$, figura 121. Dicha longitud se puede obtener mediante:
\begin{equation*}
\sum_{k=1}^n |\gamma(t_k) – \gamma(t_{k-1})| = \sum_{k=1}^n \sqrt{\left[x(t_k)-x(t_{k-1})\right]^2+\left[y(t_k)-y(t_{k-1})\right]^2}.
\end{equation*}

Entonces, la longitud de $\gamma$ es el límite, cuando este existe, de las sumas del lado derecho de la igualdad conforme la partición de $[a,b]$ se refina. Considerando lo anterior, no es difícil mostrar analíticamente que una curva suave a trozos es rectificable, es decir, el límite anterior está dado por (32.5), por lo que los detalles se dejan como ejercicio al lector.

Se puede consultar a detalle la prueba de este hecho en:

  • Introducción a funciones analíticas y transformaciones conformes, Gabriel D. Villa Salvador.
  • Function of One Complex Variable, John B. Conway.
Figura 121: Gráfica de la aproximación de la longitud de la curva $\gamma$ mediante una poligonal.

Ejemplo 32.14.
Determinemos las longitudes de las siguientes trayectorias.

a) Sean $z_1, z_2\in\mathbb{C}$ tales que $z_1 \neq z_2$. Sea $\gamma:\left[0, 1\right] \to \mathbb{C}$, dada por:
\begin{equation*}
\gamma(t) = z_1 + (z_2-z_1)t.
\end{equation*}

Solución. Geométricamente la curva asociada a $\gamma$ es el segmento de recta que une a $z_1$ y $z_2$. Más aún, es claro que $\gamma$ es una curva suave con derivada $\gamma'(t) = z_2-z_1\neq 0$, para todo $t\in[0,1]$, por lo que:
\begin{equation*}
\ell(\gamma) = \int_{0}^{1}|\gamma'(t)| \, dt = \int_{0}^{1}|z_2-z_1| \,dt = |z_2-z_1|.
\end{equation*}

b) Sean $z_0\in\mathbb{C}$ fijo y $r>0$. Sea $\gamma:\left[0, 2\pi\right] \to \mathbb{C}$, dada por:
\begin{equation*}
\gamma(t) = z_0 + re^{it}.
\end{equation*}

Solución. Geométricamente sabemos que la curva asociada a $\gamma$ es la circunferencia con centro en $z_0$ y radio $r$, la cual es una curva suave con derivada:
\begin{equation*}
\gamma'(t) = r ie^{it}\neq 0, \quad \forall t\in[0,1],
\end{equation*}por lo que:
\begin{equation*}
\ell(\gamma) = \int_{0}^{2\pi}|\gamma'(t)| \, dt = \int_{0}^{2\pi}|r ie^{it}| \, dt = \int_{0}^{2\pi} r \, dt = 2\pi r.
\end{equation*}

Tarea moral

  1. Describe las propiedades de las siguientes curvas.
    a) $\gamma(t)=t^2+it^4$, para $t\in[-1,1]$.
    b) $\gamma(t)=e^{-it^2}$, para $t\in[0,\sqrt{2\pi}]$.
    c) $\gamma(t)=2\operatorname{cos}(t)+i\operatorname{sen}(t)$, para $t\in[0,2\pi]$.
    d) $\gamma(t)=exp\left(i\pi\sqrt[3]{t}\right)$, para $t\in[-1,1]$.
    e) $\gamma(t)=e^{t}+ie^{-t}$, para $t\in[0,1]$.
  2. Determina las reglas de correspondencia de las trayectorias $-\alpha$, $-\beta$, $-\gamma$, $\alpha-\beta$, $\beta-\gamma$, $\alpha-\beta+\gamma$ y $-\alpha+\beta-\gamma$, para $\alpha(t)=t+it$, $\beta(t)=t+it^2$ y $\gamma(t)=t^2+it$, con $t\in[0,1]$. En cada caso determina la gráfica de la curva correspondiente.
  3. Sean $\gamma:[a,b]\to\mathbb{C}$ y $\beta:[c,d]\to\mathbb{C}$ dos curvas suaves a trozos tales que $\gamma(b) = \beta(c)$. Prueba que $-\gamma$ y $\gamma+\beta$ son también curvas suaves a trozos.
  4. Sean $\alpha:[a_1,a_2]\to\mathbb{C}$, $\beta:[b_1,b_2]\to\mathbb{C}$ y $\gamma:[c_1,c_2]\to\mathbb{C}$ tres curvas suaves tales que $\alpha(a_2) = \beta(b_1)$ y $\beta(b_2) = \gamma(c_1)$. Prueba que $\alpha+(\beta+\gamma) = (\alpha+\beta)+\gamma$.
  5. Sea $\gamma:[a,b]\to\mathbb{C}$ una curva y $a<c<b$. Prueba que $\gamma = \alpha+\beta$, donde $\alpha$ es la restricción de $\gamma$ al intervalo $[a,c]$ y $\beta$ es la restricción de $\gamma$ al intervalo $[c,b]$. ¿Cómo se puede utilizar este hecho para generalizar el resultado para una partición arbitraria $P : a=t_0<t_1<\cdots<t_n=b$ del intervalo $[a,b]$?
  6. Prueba que una curva suave a trozos $\gamma$ se puede expresar como la suma finita de curvas suaves.
    Hint: Utiliza el ejercicio 4.
  7. Sea $f(t)=t^2\operatorname{sen}\left(\frac{1}{t}\right)$ para $t\neq 0$ y $f(0)=0$. Prueba que la curva $\gamma(t)=t+if(t)$, con $t\in[-\pi, \pi]$, no es un contorno.
  8. Verifica que la trayectoria:
    \begin{equation*}
    \gamma : [0, 1] \to \mathbb{C}
    \end{equation*}dada por:
    \begin{equation*}
    \gamma(t) = \left\{ \begin{array}{lcc}
    3t(1+i) & \text{si} & 0 \leq t \leq \frac{1}{3},\\
    \\ 3+i-6t, & \text{si} & \frac{1}{3} \leq t \leq \frac{2}{3}, \\
    \\ (-1+i)(3-3t), & \text{si} & \frac{2}{3} \leq t \leq 1. \end{array} \right.
    \end{equation*} es otra parametrización del camino $C$ dado en el ejemplo 32.10.
  9. Demuestra la proposición 32.1.
  10. Determina la derivada de las siguientes funciones híbridas.
    a) $f(t)=te^{-it}$.
    b) $f(t)=e^{i2t^2}$.
    c) $f(t)=(2+i)\operatorname{cos}(3it)$.
    d) $f(t)=\operatorname{Log}(it)$.
    e) $f(t)=\dfrac{2+i+t}{-i-2t}$.
    f) $f(t)=\left(\dfrac{t+i}{t-i}\right)^2$.

Más adelante…

En esta primera entrada, de la cuarta unidad, hemos abordado algunos conceptos básicos pero elementales para la teoría de la integración compleja, entre ellos definimos lo que es una función híbrida, trayectoria, curva y contorno en el plano complejo $\mathbb{C}$. Vimos algunas de las propiedades de las funciones híbridas y probamos un resultado con el que ya estamos familiarizados por nuestros cursos de Cálculo, correspondiente con la regla de la cadena para la composición de funciones complejas y funciones híbridas.

En la siguiente entrada definiremos de manera formal a la integral para el caso complejo. Como veremos muchas de las definiciones que daremos estarán sustentadas en los resultados de Cálculo para las integrales de funciones reales e integrales de línea para funciones de varias variables.

Entradas relacionadas

Variable Compleja I: Funciones complejas como transformaciones. Técnicas de graficación

Por Pedro Rivera Herrera

Introducción

Sabemos que la gráfica de una función de $\mathbb{R}^2$ a $\mathbb{R}^2$ es un subconjunto de $\mathbb{R}^4$, por lo que si quisiéramos visualizar la gráfica de una función compleja de variable compleja, como hacíamos usualmente con funciones cuyas gráficas tenían lugar en $\mathbb{R}^2$ o en $\mathbb{R}^3$, nos será imposible.

Al trabajar en Cálculo con integrales dobles era de nuestro interés saber cómo se transformaban ciertas regiones $A\subset\mathbb{R}^2$ del plano, bajo ciertas transformaciones $T:\mathbb{R}^2 \to \mathbb{R}^2$ para poder entender de manera geométrica lo que sucedía al aplicar un cambio de variable con nuestras regiones de integración.

Una primera alternativa para poder estudiar la geometría de las funciones complejas, es analizar su comportamiento gráfico siguiendo esta idea de las transformaciones entre planos. Por lo que, resultará conveniente considerar a las funciones complejas como transformaciones del plano complejo, es decir, para darnos una idea de lo que hacen estas funciones las veremos como mapeos o transformaciones de un subconjunto de $\mathbb{C}$ en un plano $z$, que toman valores en un plano $w$. Si escribimos a $z=x+iy\in\mathbb{C}$ y a $w=u+iv\in\mathbb{C}$, tendremos al plano $z$ asociado con los ejes $x$ y $y$, mientras que para el plano $w$ se tienen los ejes $u$ y $v$, los cuales, en ambos casos, corresponden con los ejes real e imaginario, respectivamente.

Observación 26.1.
Debemos tener presente que esta alternativa no es la única forma para dar una interpretación del comportamiento geométrico de una función compleja, ya que también puede representarse a las funciones complejas mediante:

  1. las gráficas de su parte real e imaginaria,
  2. las gráficas del módulo complejo de la función,
  3. superficies de Riemann.

Se puede consultar un poco sobre estás técnicas en las siguientes ligas:

Sin embargo, para los fines del curso bastará con esta alternativa de pensar a las funciones como transformaciones del plano en el plano.

Para plantear lo anterior consideremos la siguiente:

Definición 26.1.
Sea $S\subset\mathbb{C}$, se define a la imagen de $S$ bajo una función $f$, denotada por $f(S)$, como el conjunto:
\begin{equation*}
f(S) = \left\{ w\in\mathbb{C} \,:\, w = f(z),\, z\in S\right\}.
\end{equation*}

Analicemos los siguientes ejemplos para comprender mejor esta idea de las transformaciones complejas.

Ejemplo 26.1.
Sea $S$ el disco unitario, es decir $S=\overline{B}(0,1)$. Determinemos la imagen de $S$ bajo la transformación $f(z) = z+2+i$.

Solución. Notemos que para cada $z\in S$ el valor de $w=f(z)$ está dado por la suma de $2+i$ al valor de $z$. Considerando a $z=x+iy$, tenemos que:
\begin{equation*}
w = f(z) = (x+2) + i(y+1),
\end{equation*}
por lo que la función $f$ transforma los puntos $(x,y)$ en los puntos $(u,v)$, donde $u=x+2$ y $v=y+1$. Es claro que $f$ simplemente traslada a cada elemento del disco unitario $S$ dos unidades a la derecha y una unidad hacia arriba, figura 98, es decir:
\begin{align*}
f(S) & = \{ w\in\mathbb{C} \,:\, |\,w-(2+i)\,|\leq 1 \}\\
& = \overline{B}(2+i,1).
\end{align*}

Figura 98: Gráfica del conjunto $S$ en el plano $z$ y del conjunto $f(S)$ en el plano $w$.

Esta transformación de la forma $f(z)=z+b$, con $b\in\mathbb{C}$ constante, nos determina una traslación. Consideremos ahora transformaciones de la forma $f(z) = az$, con $a\in\mathbb{C}\setminus\{0\}$. Veamos que dichas transformaciones nos determinan rotaciones y homotecias en el plano complejo.

Observación 26.2.
Recordemos que al multiplicar a los números complejos $a=\rho\operatorname{cis}(\alpha)\neq 0$ y $z = r \operatorname{cis}(\theta)$ tenemos:
\begin{equation*}
az = r\rho \operatorname{cis}(\alpha + \theta) = r \rho e^{i(\alpha + \theta)},
\end{equation*} de donde es claro que la transformación $f(z) = az$ nos devuelve una homotecia desde que al módulo de $z$ se le multiplica por una constante $\rho>0$. Por otra parte, notemos que tenemos una rotación del número complejo $z$ desde que a su argumento $\theta$ se le suma el argumento $\alpha$. Además, dado que el producto de números complejos es conmutativo, es claro que la homotecia y rotación que se le aplica a cada número complejo $z$ se puede realizar en cualquier orden. Notemos que tenemos los siguientes casos:

  • Si $a = \rho >0$, entonces tenemos que la transformación $f(z) = az$ es simplemente una homotecia por un factor $a$.
  • Si $\rho = 1$, entonces tenemos que la transformación $f(z) = az = e^{i\alpha} z$ es simplemente una rotación por un ángulo $\alpha$.
  • Si $\rho>0$, entonces tenemos que la transformación $f(z) = az = \rho e^{i\alpha} z$ es una homotecia por un factor $\rho$ seguida de una rotación por un ángulo $\alpha$.

Ejemplo 26.2.
Sea $S$ el cuadrado cerrado con centro en el punto $z=2$, cuyos lados son paralelos a los ejes real e imaginario y tienen una longitud $2$, figura 99.

a) ¿Cuál es la imagen de $S$ bajo la transformación $f(z)=3z$?
b) ¿Cuál es la imagen de $S$ bajo la transformación $f(z)=2iz$?

Figura 99: Gráfica del conjunto $S$ en el plano $z$.Gráfica del conjunto $S$ en el plano $z$.

Solución.

a) De acuerdo con la observación 26.2 tenemos que $a=3>0$ es una constante, por lo que la transformación $f(z)=3z$ corresponde con una homotecia, ya que bajo $f$ la distancia de cada número complejo $w=f(z)$, medida desde el origen, será tres veces la distancia de cada $z\in S$, medida desde el origen, es decir, el módulo de cada número complejo $z\in S$ será triplicado, mientras que su argumento permanecerá sin cambios. Entonces la imagen de $S$ bajo la transformación $f$, es decir, $f(S)$ será otro cuadrado cuyos vértices corresponden con las imágenes de los vértices del cuadrado $S$, figura 100.

Tenemos que el centro del cuadrado $S$ es $z=2$, mientras que sus vértices son $A=1+i$, $B=3+i$, $C=1-i$ y $D=3-i$, por lo que el centro y los vértices del nuevo cuadrado cerrado son:
\begin{align*}
f(2) = 6,\\
f(A) = 3+3i,\\
f(B) = 9+3i,\\
f(C) = 3-3i,\\
f(D) = 9-3i.
\end{align*}

Entonces $f(S)$ en el plano $w$ es el cuadrado cerrado con centro en el punto $w=6$ cuyos lados son paralelos a los ejes real e imaginario y tienen longitud 6.

Figura 100: Gráfica del conjunto $S$ en el plano $z$ y del conjunto $f(S)$ en el plano $w$.

b) Considerando la observación 26.2, tenemos que $a=2i$ es un número complejo cuyo módulo es $r=2$ y su argumento principal es $\alpha = \frac{\pi}{2}$, por lo que la transformación $f(z)=2iz$ corresponde con una homotecia por un factor $r=2$ seguida de una rotación por un ángulo $\alpha=\frac{\pi}{2}$, en sentido contrario a las manecillas del reloj. Entonces, bajo $f$ a cada punto $z\in S$ se le duplicará su módulo y se le sumará $\frac{\pi}{2}$ a su argumento, entonces $f(S)$ será nuevamente un cuadrado cerrado para el cual la distancia del origen a cada $w=f(z)$ será dos veces la distancia del origen a cada $z\in S$ y sus vértices serán las imágenes de los vértices del cuadrado $S$ bajo $f$, figura 101.

Tenemos que el centro del cuadrado $S$ es $z=2$, mientras que sus vértices son $A=1+i$, $B=3+i$, $C=1-i$ y $D=3-i$, por lo que el centro y los vértices del nuevo cuadrado cerrado son:
\begin{align*}
f(2) = 2(2)\operatorname{cis}\left(\frac{\pi}{2} + 0\right) = 4i,\\
f(A) = 2\sqrt{2}\operatorname{cis}\left(\frac{\pi}{2} + \frac{\pi}{4}\right) = -2+2i,\\
f(B) = 2\sqrt{10}\operatorname{cis}\left(\frac{\pi}{2} + \operatorname{arc tan}\left(\frac{1}{3}\right)\right) = -2+6i,\\
f(C) = 2\sqrt{2}\operatorname{cis}\left(\frac{\pi}{2} – \frac{\pi}{4}\right) = 2+2i,\\
f(D) = 2\sqrt{10}\operatorname{cis}\left(\frac{\pi}{2} + \operatorname{arc tan}\left(\frac{-1}{3}\right)\right) = 2+6i.
\end{align*}

Entonces $f(S)$ en el plano $w$ es el cuadrado cerrado con centro en el punto $w=4i$ cuyos lados son paralelos a los ejes real e imaginario y tienen longitud 4.

Figura 101: Gráfica del conjunto $S$ en el plano $z$ y del conjunto $f(S)$ en el plano $w$.

Es posible visualizar las transformaciones de los ejemplos 26.1 y 26.2 en el siguiente applet de GeoGebra https://www.geogebra.org/m/pasmamyw.

Observación 26.3.
En conjunto, los ejemplos 26.1 y 26.2 corresponden con transformaciones del tipo $f(z) = az + b$, donde $a,b\in\mathbb{C}$ son dos constantes, y $a\neq 0$ para no trabajar con transformaciones constantes, es decir, transformaciones afines lineales, definición 25.1.

Más aún, de acuerdo con dichos ejemplos, debe ser claro que las transformaciones afines lineales mapean regiones del plano a regiones geométricamente similares. En el caso en que $a=1$ y $b=0$, entonces tenemos a la transformación identidad $\mathbb{I}_\mathbb{C}(z)=z$.

Una pregunta interesante es ¿qué sucede con las transformaciones que no son lineales?, es decir ¿en qué se transforman los subconjuntos de $\mathbb{\mathbb{C}}$ bajo una transformación no lineal? Para responder a esta pregunta analicemos los siguientes ejemplos.

Ejemplo 26.3.
Determinemos la imagen de los siguientes conjuntos bajo la transformación inversión, $f(z) = \dfrac{1}{z}$, con $z\neq 0$, $w=f(z)\neq 0$.

a) $S = \left\{z\in\mathbb{C} \,:\, 0<|\,z\,|<1, 0\leq \operatorname{arg} z \leq \dfrac{\pi}{2}\right\}.$
b) $S = \left\{z\in\mathbb{C} \,:\, 2\leq |\,z\,|, 0\leq \operatorname{arg} z \leq \pi\right\}.$

Solución. De acuerdo con el corolario 4.1 (fórmula de De Moivre), para $z = r\operatorname{cis}(\theta)\neq 0$, con $r=|\,z\,|$ y $\theta = \operatorname{arg} z$, tenemos que:
\begin{equation*}
f(z) = \frac{1}{z} = r^{-1}\operatorname{cis}(\theta) = \frac{1}{r}\left[\operatorname{cos}(-\theta) + i\,\operatorname{sen}(-\theta) \right].
\end{equation*}

Entonces, para cada $z\neq 0$, se tiene que el módulo de $f(z)$ es el recíproco del módulo de $z$, mientras que el argumento de $f(z)$ será el negativo del argumento de $z$.

a) Notemos que si $z\in S$, entonces $z$ cae en el primer cuadrante dentro de la circunferencia unitaria, incluyendo a los ejes real e imaginario, pero sin considerar a $z=0$, figura 102, ya que:

  • Si $0<|z|<1$, entonces $z$ cae dentro del disco unitario perforado, es decir $z\in B^*(0,1)$.
  • Si $0\leq \theta \leq \dfrac{\pi}{2}$, entonces tenemos a los $z$ en el primer cuadrante.

Por lo que bajo $f$, los $z\in S$ serán mapeados, en el plano $w$, fuera del disco unitario y en el cuarto cuadrante, incluyendo a los ejes real e imaginario, pero a partir de los $w$ tales que $|w|>1$, ya que $|\,f(z)\,| = \dfrac{1}{|\,z\,|} > 1$ y $-\frac{\pi}{2}\leq -\theta \leq 0$. Entonces:
\begin{equation*}
f(S) = \left\{ w\in\mathbb{C} \, : \, 1<|\,w\,|, -\frac{\pi}{2}\leq \operatorname{arg} w \leq 0 \right\}.
\end{equation*}

b) Notemos que si $z\in S$, entonces $z$ cae en el primer y segundo cuadrante fuera del disco de radio 2 con centro en el origen, incluyendo a la circunferencia de radio $2$ y a los ejes real e imaginario, figura 103, ya que:

  • Si $|z|\geq 2$, entonces $z$ cae sobre la circunferencia de radio $2$ y fuera de la misma.
  • Si $0\leq \theta \leq \pi$, entonces tenemos a los $z$ en el primer y segundo cuadrante.

Como el módulo de $z$ crece de $2$ a infinito, entonces bajo $f$ el módulo de $w=f(z)$ decrece de $\dfrac{1}{2}$ a $0$, pero sin llegar a valer $0$, ya que $|\,f(z)\,| = \dfrac{1}{|\,z\,|} \leq \dfrac{1}{2}$ y $|\,f(z)\,| >0$ para $z\neq 0$. Por otra parte, como $-\pi \leq – \theta \leq 0$, entonces bajo $f$ los $z$ serán mapeados, en el plano w, en el tercer y cuarto cuadrante en el disco cerrado con centro en el origen y radio $\dfrac{1}{2}$, por lo que:
\begin{equation*}
f(S) = \left\{ w\in\mathbb{C} \, : \, 0<|\,w\,| \leq \frac{1}{2}, -\pi\leq \operatorname{arg} w \leq 0 \right\}.
\end{equation*}

Figura 102: Gráfica del conjunto $S$ en el plano $z$ y del conjunto $f(S)$ en el plano $w$.
Figura 103: Gráfica del conjunto $S$ en el plano $z$ y del conjunto $f(S)$ en el plano $w$.

Ejemplo 26.4.
Sean $a,b\in\mathbb{R}$ tales que $0<a<b$. Veamos cuál es la imagen de los siguientes conjuntos bajo la transformación inversión, $T(z)=\dfrac{1}{z}$.
a) La recta vertical $x = x_0 > 0$ en el plano $z$, es decir $S = \left\{z=x+iy\in\mathbb{C} \,:\, x=x_0>0, y\in\mathbb{R}\right\}$.
b) La franja vertical $S = \left\{z=x+iy\in\mathbb{C} \,:\, a\leq x \leq b, y\in\mathbb{R}\right\}$, en el plano $z$.
\end{itemize}

Solución.

a) De acuerdo con la observación 25.8 como la recta vertical $x=x_0>0$ no pasa por el origen, entonces la imagen de $S$ bajo $T$ será una circunferencia que pase por el origen en el plano $w$.

Sea $z=x+iy \in S$, entonces tenemos $x = x_0 > 0$ y $y\in\mathbb{R}$. La imagen de $z$ bajo $T$ esta dada por:
\begin{equation*}
w = u+iv = \frac{1}{z}.
\end{equation*}

De acuerdo con (25.7), proposición 25.6, tenemos que:
\begin{equation*}
u=\frac{x_0}{x_0^2+y^2}, \quad v = -\frac{y}{x_0^2+y^2},
\end{equation*}de donde:
\begin{equation*}
u^2 +v^2 = \frac{x_0^2 +y^2}{\left(x_0^2+y^2\right)^2} = \frac{1}{x_0^2+y^2} = \frac{u}{x_0},
\end{equation*}por lo que:
\begin{align*}
&u^2 -\frac{u}{x_0} + v^2 = 0,\\
& \Longleftrightarrow u^2 -\frac{u}{x_0} + \left(-\frac{1}{2x_0}\right)^2 + v^2 = \left(-\frac{1}{2x_0}\right)^2,\\
& \Longleftrightarrow \left( u -\frac{1}{2x_0}\right)^2 + v^2 = \left(\frac{1}{2x_0}\right)^2, \tag{26.1}
\end{align*}la cual corresponde con la ecuación de una circunferencia con centro en el punto $w_0 = \dfrac{1}{2x_0} + i0$ y radio $r=\dfrac{1}{2x_0}$, en el plano $w$. Además, notemos que mientras el punto $z=0$ no cae en la recta $S$, el punto $w=0$ sí pertenece a $T(S)$.

b) Considerando el inciso anterior y la observación 25.8, si $0<a<x_0<b$, entonces la recta vertical $x=x_0$ recorre la franja vertical $S$ en el plano $z$, por lo que la imagen de cada recta vertical $x=x_0$ bajo la transformación inversión corresponderá con una circunferencia dada por (26.1), en el plano $w$. De acuerdo con la figura 104 tenemos que $T(S)$ es la región anular acotada por fuera por la circunferencia centrada en $\left(\dfrac{1}{2a}, 0\right)$ de radio $\dfrac{1}{2a}$ y por dentro por la circunferencia centrada en $\left(\dfrac{1}{2b}, 0\right)$ de radio $\dfrac{1}{2b}$.

Figura 104: Gráfica de la franja vertical $S$ bajo la transformación inversión.

Ejemplo 26.5.
Determinemos la imagen de la recta vertical $x=k$, donde $k\in\mathbb{C}$ es una constante, bajo la transformación $f(z)=z^2$.

Solución. Sean $z=x+iy\in\mathbb{C}$ y $w=u+iv = f(z)$, entonces:
\begin{equation*}
u(x,y)=x^2-y^2, \quad v(x,y)=2xy.
\end{equation*}

Dado que $x=k$, entonces los puntos sobre dicha recta vertical son de la forma:
\begin{equation*}
z=k+iy, \quad y\in\mathbb{R},
\end{equation*}por lo que, la imagen de dicha recta, bajo $f$, es:
\begin{equation*}
u=k^2-y^2, \quad v=2ky, \quad y\in\mathbb{R} \tag{26.2}.
\end{equation*}

Si $k=0$, entonces la imagen de la recta vertical $x=0$, correspondiente con el eje imaginario, está dada por:
\begin{equation*}
u=-y^2, \quad v=0, \quad y\in\mathbb{R}.
\end{equation*}

Es decir, la imagen del eje imaginario, bajo la función $f(z)=z^2$, corresponde con el semieje real negativo.

Por otra parte, si $k\neq 0$, entonces de (26.2) tenemos que:
\begin{equation*}
y = \frac{v}{2k} \quad \Longrightarrow \quad u = k^2 – \frac{v^2}{4k^2}, \quad \Longrightarrow \quad v^2 = -4k^2(u – k^2), \quad v\in\mathbb{R}.
\end{equation*}

Por lo que, para $k\neq0$, la imagen de la recta $x=k$ corresponde con la familia de parábolas con eje paralelo al eje real $u$, con vértice en el punto $\left(k^2,0\right)$, las cuales abren hacia la izquierda y cuya intersección con el eje imaginario $v$ son los puntos $\left(0, \pm 2k^2\right)$. Dado que la imagen de las rectas $x=k$ y $x=-k$ es la misma, entonces ambas rectas, bajo $f$, son mapeadas en la parábola $v^2 = -4k^2(u – k^2)$.

Podemos visualizar lo anterior en el siguiente applet de GeoGebra https://www.geogebra.org/m/y75hxksq.

Observación 26.4.
De manera general, se puede hacer un análisis para la transformación $f(z)=z^n$, con $n>2$. Si consideramos a $z\neq 0$ tal que $z=re^{i\theta}$ y $w=\rho e^{i\phi}$, entonces:
\begin{equation*}
w=f(z) \quad \rho e^{i\phi} = r^n e^{in\theta}.
\end{equation*}

No es difícil verificar que dicha transformación mapea la región:
\begin{equation*}
\left\{z=re^{i\theta}\in\mathbb{C}\setminus\{0\} : r\geq 0, \, 0\leq \theta \leq \frac{\pi}{n}\right\},
\end{equation*}en el semiplano superior del plano $w$.

Ejemplo 26.6.
Consideremos a la región rectangular:
\begin{equation*}
S=\left\{z=x+iy\in\mathbb{C} : -1\leq x\leq 1, 0\leq y\leq \pi\right\}
\end{equation*}y determinemos su imagen bajo la transformación $f(z) = e^z$.

Solución. Fijemos a $x_0\in[-1,1]$ y consideremos al segmento de la recta vertical $x=x_0$ que está completamente contenido en $S$. Los puntos de dicho segmento son de la forma $z=x_0+iy$, con $0\leq y \leq \pi$. Para dichos puntos, por la proposición 20.2, tenemos que:
\begin{equation*}
w = f(z) = e^{x_0+iy} = e^{x_0} e^{iy} = e^{x_0}\left[\operatorname{cos}(y) + i \operatorname{sen}(y)\right], \quad 0\leq y \leq \pi,
\end{equation*}de donde se sigue que los puntos $w=e^{x_0}\operatorname{cis}(y)$ tienen módulo $e^{x_0}$ y argumento principal $y$.

Notemos que los puntos $w=f(z)$ están sobre la circunferencia de radio $e^{x_0}$ y centro en el origen. Más aún, como $y\in[0,\pi]$, entonces dichos puntos trazan la semicircunferencia ubicada en el semiplano superior del plano $w$.

Dado que $x_0\in[-1,1]$, entonces $e^{-1}<e^1$, por lo que, los segmentos de las rectas verticales $x=x_0$, contenidos en $S$, corresponden con circunferencias de radio creciente y en conjunto forman la región anular delimitada por las semicircunferencias de radio $e^{-1}$ y $e^1$, respectivamente, amabas centradas en el origen.

De manera análoga, podemos verificar que bajo la transformación $f(z)=e^z$, la región rectangular:
\begin{equation*}
\left\{z=x+iy\in\mathbb{C} : -1\leq x\leq 1, -\pi\leq y\leq 0\right\},
\end{equation*}es mapeada en la región anular delimitada por las mismas semicircunferencias de radio $e^{-1}$ y $e^1$, con centro en el origen, ubicada en el semiplano inferior del plano $w$.

En general, si consideramos a la región fundamental de la exponencial, figura 78 entrada 20, es decir, la banda infinita:
\begin{equation*}
\left\{z=x+iy\in\mathbb{C} : -\infty <x<\infty, -\pi< y < \pi\right\},
\end{equation*}entonces, dicho conjunto es mapeado bajo la transformación $f(z)=e^z$ en el conjunto:
\begin{equation*}
\mathbb{C}\setminus{L_{-\pi}} = \left\{z\in\mathbb{C} : |z|>0, -\pi< \operatorname{Arg}(z) < \pi\right\},
\end{equation*}ya que $\lim\limits_{x\to-\infty} e^x =0$ y $\lim\limits_{x\to\infty} e^x = \infty$.

Podemos visualizar lo anterior en el siguiente applet de GeoGebra https://www.geogebra.org/m/qqyhwmgv.

Ejemplo 26.7.
Sean $0<a<b$ y $0\leq \theta_1 <\theta_2 \leq \pi$. Determinemos la imagen de la región circular:
\begin{equation*}
S = \left\{z\in\mathbb{C} : a \leq |z| \leq b,\,\, \theta_1 \leq \operatorname{Arg}(z) \leq \theta_2\right\},
\end{equation*}bajo la transformación $\operatorname{Log}(z)$.

Solución. Notemos que la región $S$ está delimitada por las semirrectas que parten del origen y se forman por los ángulos $0\leq\theta_1$, $\theta_2\leq\pi$ y por los arcos de circunferencia de radio $a$ y $b$ con centro en el origen, como se puede ver en el siguiente applet de GeoGebra https://www.geogebra.org/m/qjzxhefv.

Consideremos a una semirrecta $L$ que parte del origen y está determinada por un ángulo $\theta$ tal que $\theta_1 \leq \theta \leq \theta_2$. Sea $z\in L \cap S$, entonces:
\begin{align*}
\operatorname{Log}(z) & = \operatorname{ln}|z| +i \operatorname{Arg}(z)\\
& = \operatorname{ln}|\,z\,| +i \theta.
\end{align*}

Dado que $0<a\leq |\,z\,| \leq b$, tenemos que $\operatorname{ln}(a) \leq \operatorname{ln}|\,z\,| \leq \operatorname{ln}(b)$, por lo que los puntos $w=\operatorname{Log}(z)$ describen el segmento de la recta horizontal:
\begin{equation*}
w=u+iv, \quad \operatorname{ln}(a) \leq u \leq \operatorname{ln}(b), \quad v=\theta.
\end{equation*}

Como $v\in[\theta_1, \theta_2]$, entonces el segmento de la semirrecta $L$, que está completamente contenido en $S$, cubre a $S$ conforme $v$ varía, por lo que bajo $f$ dicho segmento cubre a la región rectangular en el plano $w$ determinada por los vértices $\left(\operatorname{ln}(a), \theta_1\right)$, $\left(\operatorname{ln}(b), \theta_1\right)$, $\left(\operatorname{ln}(b), \theta_2\right)$ y $\left(\operatorname{ln}(a), \theta_2\right)$.

Observación 26.5.
De acuerdo con los ejemplos 26.6 y 26.7, geométricamente debe ser claro que las funciones $\operatorname{Log}(z)$ y $e^z$ biyectan una región anular en una región rectangular y viceversa. Además, la frontera de la región anular es mapeada en la frontera de la región rectangular y viceversa.

Cerraremos esta entrada con el siguiente ejemplo correspondiente con la transformación trigonométrica $\operatorname{sen}(z)$. Por simplicidad consideraremos solo una parte de una banda infinita, pues para la parte restante el planteamiento es el mismo, sin embargo no deja de ser de suma importancia pues en conjunto nos permiten concluir que la función compleja $\operatorname{sen}(z)$ tiene como imagen a todo el plano complejo $\mathbb{C}$.

Ejemplo 26.8.
Determinemos la imagen de la semibanda infinita:
\begin{equation*}
S = \left\{z=x+iy\in\mathbb{C} : -\frac{\pi}{2}\leq x\leq \frac{\pi}{2},\, y\geq 0\right\},
\end{equation*}bajo la transformación $f(z)=\operatorname{sen}(z)$.

Solución. De manera análoga a los ejemplos anteriores, procedemos a cubrir la región del dominio de $f$ con alguna curva simple que bajo $f$ nos permita cubrir la imagen de $S$ y así determinar dicho conjunto.

Sea $0\leq y_0 <\infty$ fijo. Consideramos el segmento de recta horizontal contenido en $S$ dado por:
\begin{equation*}
y=y_0, \quad -\frac{\pi}{2}\leq x\leq \frac{\pi}{2}.
\end{equation*}

Sea $z=x+iy_0$ un punto sobre dicho segmento de recta, entonces, bajo $f$, por la proposición 22.1(10) tenemos que:
\begin{align*}
w=u+iv & = \operatorname{sen}(x+iy_0)\\
& = \operatorname{sen}(x) \operatorname{cosh}(y_0) + i \operatorname{cos}(x) \operatorname{senh}(y_0),
\end{align*}de donde:
\begin{equation*}
u(x,y) = \operatorname{sen}(x) \operatorname{cosh}(y_0) \quad \text{y} \quad v(x,y) = \operatorname{cos}(x) \operatorname{senh}(y_0).
\end{equation*}

Si $y_0=0$, tenemos que $\operatorname{cosh}(0)=1$ y $\operatorname{senh}(0)=0$, por lo que $v=0$ y $u=\operatorname{sen}(x)$, entonces la imagen del intervalo $\left[-\frac{\pi}{2}, \frac{\pi}{2}\right]$, bajo $f(z)=\operatorname{sen}(z)$, es el intervalo $[-1,1]$.

Supongamos ahora que $y_0>0$. Tenemos que $\operatorname{cosh}(y_0)>0$ y $\operatorname{senh}(y_0)>0$, por lo que:
\begin{equation*}
\operatorname{sen}(x) = \frac{u}{\operatorname{cosh}(y_0)} \quad \text{y} \quad \operatorname{cos}(x) = \frac{v}{\operatorname{senh}(y_0)} \tag{26.3}.
\end{equation*}

Para $x\in\left[-\frac{\pi}{2}, \frac{\pi}{2}\right]$ tenemos que $\operatorname{cos}(x)\leq 0$, por lo que $v\geq 0$.

Elevando al cuadrado ambas igualdades en (26.3) y sumándolas tenemos que:
\begin{equation*}
1 = \operatorname{sen}^2(x) + \operatorname{cos}^2(x) = \left(\frac{u}{\operatorname{cosh}(y_0)}\right)^2 + \left(\frac{v}{\operatorname{senh}(y_0)}\right)^2. \tag{26.4}
\end{equation*}

De acuerdo con (26.4), como $x\in\left[-\frac{\pi}{2}, \frac{\pi}{2}\right]$, entonces los puntos $w=u+iv$ trazan la semielipse superior:
\begin{equation*}
\left(\frac{u}{\operatorname{cosh}(y_0)}\right)^2 + \left(\frac{v}{\operatorname{senh}(y_0)}\right)^2 = 1, \quad v\geq 0.
\end{equation*}

Los puntos de intersección de dicha semielipse con el eje real $u$ son $\left(\pm\operatorname{cosh}(y_0), 0\right)$, mientras que el punto intersección con el eje imaginario $v$ es $\left(0,\operatorname{senh}(y_0)\right)$.

Dado que:
\begin{align*}
\lim\limits_{y_0\to\infty} \operatorname{senh}(y_0) = \infty, &\quad \lim\limits_{y_0\to\infty} \operatorname{cosh}(y_0) = \infty,\\
\lim\limits_{y_0\to 0 } \operatorname{senh}(y_0) =0, &\quad \lim\limits_{y_0\to 0} \operatorname{cosh}(y_0) = 1,
\end{align*}y $y_0\in(0,\infty)$, entonces la imagen de las semielipses se encuentra en el semiplano superior $v\geq 0$, del plano $w$, incluyendo el eje real $u$.

Se puede verificar fácilmente, ejercicio 5, que la frontera de $S$ es mapeada en la frontera de $f(S)$, correspondiente con el eje real $u$.

Podemos visualizar lo anterior en el siguiente applet de GeoGebra https://www.geogebra.org/m/byzgzgzb.

Tarea moral

  1. Determina la imagen $f(S)$ bajo la transformación lineal dada.
    a) $f(z) = 4z$ y $S=\left\{ z\in\mathbb{C} : |z|<1 \right\}$.
    b) $f(z) = iz+i$ y $S=\left\{ z\in\mathbb{C} : \operatorname{Re}(z)>0 \right\}$.
    c) $f(z) = -z+2i$ y $S=\left\{ z\in\mathbb{C} : \operatorname{Re}(z)>0, \operatorname{Im}(z)>0 \right\}$.
    d) $f(z) = iz+2$ y $S=\left\{ z\in\mathbb{C} : |z|\leq 2, 0\leq \operatorname{Arg}(z)\leq \dfrac{\pi}{2} \right\}$.
  2. Sea $f(z)=\dfrac{1}{z}$ la transformación inversión. Para cada conjunto $S$ determina su imagen $f(S)$.
    a) $S=\left\{ z\in\mathbb{C} : 0<|z|\leq 1 \right\}$.
    b) $S=\left\{ z\in\mathbb{C} : |z|\geq 1 \right\}$.
    c) $S=\left\{ z\in\mathbb{C} : 0<|z|\leq 3, \dfrac{\pi}{3}\leq \operatorname{Arg}(z)\leq \dfrac{2\pi}{3} \right\}$.
    d) $S=\left\{ z\in\mathbb{C} : z\neq 0, 0 \leq \operatorname{Arg}(z)\leq \dfrac{\pi}{2} \right\}$.
  3. Encuentra la imagen de $S=\left\{ z\in\mathbb{C} : |z|<1 \right\}$ bajo $f(z)=z+\overline{z}$.
  4. Sea $f(z) = z^2$. Determina la imagen $f(S)$ de cada conjunto $S$.
    a) $S$ es el cuadrado con vértices $(0,0), (1,0), (1,1)$ y $(0,1)$.
    b) $S=\left\{ z\in\mathbb{C} : 0\leq \operatorname{Im}(z) \leq 1 \right\}$.
    c) $S=\left\{ z\in\mathbb{C} : \operatorname{Re}(z)>0, \operatorname{Im}(z)>0 \right\}$.
    d) $S=\left\{ z\in\mathbb{C} : -2 \leq \operatorname{Re}(z) \leq 0 \right\}$.
  5. Sea $f(z) = \operatorname{sen}(z)$.
    a) Muestra que bajo $f$ la semirrecta $x=\dfrac{\pi}{2}$, $y\geq 0$ es mapeada en la semirrecta $u\geq 1$, $v = 0$.
    b) Muestra que bajo $f$ la semirrecta $x=-\dfrac{\pi}{2}$, $y\geq 0$ es mapeada en la semirrecta $u\leq -1$, $v = 0$.
    c) Concluye que la frontera del conjunto $S$ en el ejemplo 26.8 es mapeada en la drontera del conjunto $f(S)$.
    d) Sabemos que la elipse:
    \begin{equation*}
    \dfrac{x^2}{a^2} + \dfrac{y^2}{b^2} = 1,
    \end{equation*}con $0<b<a$, tiene focos en los puntos $\left(\pm \sqrt{a^2 – b^2}, 0\right)$. Muestra que todas las elipses del ejemplo 26.8 tienen los mismos focos en los puntos $(\pm 1,0)$.
  6. Sea $S$ la semibanda horizontal:
    \begin{equation*}
    \left\{z=x+iy\in\mathbb{C} : x\geq 0, -\frac{\pi}{2} \leq y \leq \frac{\pi}{2} \right\}.
    \end{equation*}Determina la imagen de $S$ bajo la función $f(z)=\operatorname{senh}(z)$.

    Hint: Expresa a $\operatorname{senh}(z)$ en términos de $\operatorname{sen}(z)$.
  7. Sea $w=\rho e^{i\phi}$. Muestra que la transformación $w=\dfrac{i}{z}$ mapea la hipérbola $x^2-y^2=1$ en la lemniscata $\rho^2 = 2 \operatorname{cos}(2\phi)$.
  8. Determina una transformación que rote a la elipse $x^2+xy+y^2=2$, en el sentido de las manecillas del reloj, tal que su ecuación se reduzca a su forma canónica. Obtén la longitud de su semieje mayor y menor.

    Hint: Supón que la transformación es de la forma $w=\rho e^{i\phi}$. Determina un ángulo $\alpha$ tal que el coeficiente de $uv$ en la imagen de la curva es cero.

Más adelante…

En esta entrada hemos abordado, a manera de ejemplos, el estudio de las funciones complejas como transformaciones del plano complejo, esto con el objetivo de dar una interpretación geométrica del comportamiento de las funciones complejas, ya que como mencionamos antes, visualizar la gráfica de tales funciones resulta imposible. Debe ser claro que la propuesta de esta entrada es solo una de las distintas alternativas conocidas para el estudio de la parte gráfica de una función compleja, sin embargo hay otras alternativas que pueden ser de utilidad para la comprensión del comportamiento geométrico de estas funciones, por lo que se recomienda consultar estas otras propuestas.

Con esta entrada finalizamos la segunda unidad del curso. En la siguiente entrada es la primera de la tercera unidad del curso, correspondiente con el tema de series de números complejos, en la cual estudiaremos algunos de los conceptos básicos así como algunas de las propiedades más importantes de estos objetos matemáticos, mediante los cuales probaremos una serie de resultados que serán de utilidad para caracterizar a las funciones complejas a través de dichos objetos.

Entradas relacionadas

Variable Compleja I: Funciones inversas de las funciones trigonométricas e hiperbólicas complejas

Por Pedro Rivera Herrera

Introducción

La entrada anterior definimos a las funciones complejas trigonométricas e hiperbólicas, a través de la exponencial compleja y vimos que tanto las funciones trigonométricas como las funciones hiperbólicas son periódicas, puesto que la función exponencial compleja es $2\pi i $-periódica. Al igual que en el caso real, podemos preguntarnos si es que existen las funciones inversas de estas funciones, por lo que nuestro objetivo en esta entrada será responder esa pregunta y en particular deducir y definir a las funciones inversas de dichas funciones.

Durante esta entrada utilizaremos nuevamente al logaritmo complejo para deducir a las funciones inversas. Es importante recordar, entrada 21, que la función logaritmo complejo es una función multivaluada, por lo que las funciones definidas en esta entrada serán también multivaluadas. Recordemos que aunque esta terminología no cumple con la definición habitual de función, desde que asigna más de un valor a cada elemento del dominio, es importante mencionar que cada una de las ramas de dichas funciones multivaluadas sí cumplen con la definición de función con la que estamos familiarizados. Más aún, cada una de las ramas cumple con muchas de las propiedades que conocemos para sus versiones reales.

Observación 23.1.
Como veremos en la entrada 26, la imagen de las funciones $\operatorname{sen}(w)$ y $\operatorname{cos}(w)$ es todo el plano complejo $\mathbb{C}$, por lo que dado $w\in\mathbb{C}$ siempre existirá $z\in\mathbb{C}$ que satisfaga $z = \operatorname{sen}(w)$ ó $z = \operatorname{cos}(w)$.

Proposición 23.1 (Funciones trigonométricas inversas.)

  1. $\operatorname{sen}^{-1}(z) = -i \operatorname{log}\left(iz +\sqrt{1-z^2}\right)$.
  2. $\operatorname{cos}^{-1}(z) = -i \operatorname{log}\left(z + i\sqrt{1-z^2}\right)$.
  3. $\operatorname{tan}^{-1}(z) = \dfrac{i}{2} \operatorname{log}\left(\dfrac{i+z}{i-z}\right)$, con $z\neq \pm i$.
  4. $\operatorname{cot}^{-1}(z) = -\dfrac{i}{2} \operatorname{log}\left(\dfrac{z+i}{z-i}\right)$, con $z\neq \pm i$.
  5. $\operatorname{sec}^{-1}(z) = \dfrac{1}{i} \operatorname{log}\left(\dfrac{1 + \sqrt{1-z^2}}{z}\right)$.
  6. $\operatorname{csc}^{-1}(z) = \dfrac{1}{i} \operatorname{log}\left(\dfrac{1 + \sqrt{z^2 – 1}}{z}\right)$.

Demostración.

  1. Sea $z = \operatorname{sen}(w)$. De acuerdo con la definición 22.1 tenemos que:
    \begin{equation*}
    z = \operatorname{sen}(w) = \frac{e^{iw} – e^{-iw}}{2i},
    \end{equation*}de donde:
    \begin{equation*}
    (e^{iw})^2-i2z(e^{iw}) – 1 = 0.
    \end{equation*}Notemos que esta última expresión es una ecuación cuadrática para la variable $e^{iw}$, por lo que podemos utilizar la fórmula general para resolver dicha ecuación. En este punto es importante que recordemos que la función compleja raíz cuadrada es una función multivaluada, por lo que nos dará dos raíces complejas, entonces:
    \begin{equation*}
    e^{iw} = \frac{2iz + \sqrt{4(1-z^2)}}{2} = iz + \sqrt{1-z^2}.
    \end{equation*}Dependiendo de la rama que consideremos, la función $ \sqrt{1-z^2}$ nos determina dos raíces cuadradas de $1-z^2$. Estableciendo la rama de la función multivaluada $ \sqrt{1-z^2}$ con la que trabajaremos, podemos utilizar la proposición 20.1(6) obteniendo:
    \begin{equation*}
    iw = \operatorname{log}\left(iz + \sqrt{1-z^2}\right) + i2k\pi, \quad k\in\mathbb{Z}.
    \end{equation*}Por lo que, para $k, n\in\mathbb{Z}$:
    \begin{align*}
    w &= \frac{1}{i}\operatorname{log}\left(iz + \sqrt{1-z^2}\right) + 2 k\pi\\
    &= \frac{1}{i} \left[\operatorname{ln}\left|iz + \sqrt{1-z^2}\right| + i\left(\operatorname{Arg}\left(iz + \sqrt{1-z^2}\right) + 2n\pi\right)\right] + 2 k\pi\\
    &= \frac{1}{i} \left[\operatorname{ln}\left|iz + \sqrt{1-z^2}\right| + i\left(\operatorname{Arg}\left(iz + \sqrt{1-z^2}\right) + 2m\pi\right)\right]\\
    &= \frac{1}{i} \left[\operatorname{ln}\left|iz + \sqrt{1-z^2}\right| + i\operatorname{arg}\left(iz + \sqrt{1-z^2}\right)\right]\\
    &= -i \operatorname{log}\left(iz + \sqrt{1-z^2}\right),
    \end{align*}donde $m=k+n\in\mathbb{Z}$.
  2. Se deja como ejercicio al lector
  3. Se deja como ejercicio al lector
  4. Sea $z = \operatorname{cot}(w)$. De acuerdo con la definición 22.2 sabemos que:
    \begin{equation*}
    \operatorname{cot}(w) = \frac{\operatorname{cos}(w)}{\operatorname{sen}(w)},
    \end{equation*}donde $w \neq k \pi$, con $k\in\mathbb{Z}$.

    Considerando lo anterior tenemos que:
    \begin{equation*}
    z = \operatorname{cot}(w) = i \left( \frac{e^{iw} + e^{-iw}}{e^{iw} – e^{-iw}}\right) = i \left( \frac{e^{i2w} + 1}{e^{i2w} – 1}\right),
    \end{equation*}de donde:
    \begin{equation*}
    -iz = \frac{e^{i2w} + 1}{e^{i2w} – 1}\quad \Longrightarrow e^{i2w} = \frac{iz – 1}{iz + 1} = \frac{z + i}{z – i}.
    \end{equation*}De acuerdo con la proposición 20.1(6) tenemos que:
    \begin{equation*}
    i2w = \operatorname{log}\left(\frac{z + i}{z – i}\right) + i2k \pi, \quad k\in\mathbb{Z}.
    \end{equation*}Entonces, para $k,n\in\mathbb{Z}$:
    \begin{align*}
    w & = \frac{1}{2i} \operatorname{log}\left(\frac{z + i}{z – i}\right) + k \pi\\
    & = \frac{1}{2i} \left[\operatorname{ln}\left|\frac{z + i}{z – i}\right| + i \left(\operatorname{Arg}\left(\frac{z+i}{z-i}\right) + 2\pi n\right) \right] + k \pi\\
    & = \frac{1}{2i} \left[\operatorname{ln}\left|\frac{z + i}{z – i}\right| + i \left(\operatorname{Arg}\left(\frac{z+i}{z-i}\right) + 2\pi m\right) \right]\\
    & = \frac{1}{2i} \left[\operatorname{ln}\left|\frac{z + i}{z – i}\right| + i \operatorname{arg}\left(\frac{z+i}{z-i}\right)\right]\\
    & = -\frac{i}{2} \operatorname{log}\left(\frac{z + i}{z – i}\right),
    \end{align*}donde $m=n+k \in\mathbb{Z}$.
  5. Se deja como ejercicio al lector.
  6. Se deja como ejercicio al lector

$\blacksquare$

Observación 23.2.
Puesto que todas las funciones inversas, de las funciones trigonométricas, están dadas en términos de la función multivaluada logaritmo, entonces también dichas funciones son multivaluadas. Más aún, de acuerdo con la proposición 23.1, debe ser claro que se puede elegir una rama de alguna de estas funciones eligiendo primero una rama de la función multivaluada raíz cuadrada y luego una rama adecuada del logaritmo de modo que la función en cuestión esté bien definida.

Ejemplo 23.1.
Supongamos que $z$ es un número real en el intervalo $(-1,1)$. Veamos que si utilizamos las ramas principales de las funciones multivaluadas raíz cuadrada y logaritmo complejo entonces obtenemos la rama principal de la función inversa de la función real seno.

Solución. Sean $z=x+iy\in\mathbb{C}$. Consideremos a la funciones multivaluadas:
\begin{equation*}
F(z) = \operatorname{log}(z) \quad \text{y} \quad G(z) = \sqrt{1-z^2} = e^{\frac{1}{2} \operatorname{Log}(1-z^2)} e^{ik\pi}, \,\, k=0,1.
\end{equation*}

Para la primera función tenemos que su corte de rama está dado por los $z=x+iy\in\mathbb{C}$ tales que:
\begin{equation*}
-\pi < \operatorname{Arg}(z) \leq \pi \quad \Longleftrightarrow \quad \left\{ \begin{array}{l}
\operatorname{Re}(z) = x \leq 0, \\
\\ \operatorname{Im}(z) = y = 0.
\end{array}
\right.
\end{equation*}

De manera análoga, para la segunda función tenemos que su corte de rama está dado por los $z=x+iy\in\mathbb{C}$ tales que:
\begin{equation*}
\left\{ \begin{array}{l}
\operatorname{Re}(1-z^2) = 1-x^2+y^2 \leq 0, \\
\\ \operatorname{Im}(1-z^2) = -2xy = 0.
\end{array}
\right.
\end{equation*}

Como trabajaremos con las ramas principales de ambas funciones y $z=x\in\mathbb{R}$, entonces los cortes de rama de cada función, son respectivamente:
\begin{equation*}
\left\{x\in\mathbb{R} : x\leq 0\right\} \quad \text{y} \quad \left\{x\in\mathbb{R} : |x|\geq 1\right\}.
\end{equation*}

Para $k=0$ tenemos de la segunda función que su rama principal es:
\begin{equation*}
g_0(z) := \sqrt{1-z^2} = e^{\frac{1}{2} \operatorname{Log}(1-z^2)}.
\end{equation*}

Como $|z|=|x|<1$, entonces la función $g_0$ está bien definida, más aún, tenemos que $ 0<1-z^2\leq 1$, por lo que $\operatorname{Arg}(1-z^2) = 0$, entonces:
\begin{equation*}
\operatorname{Log}(1-z^2) = \operatorname{ln}|1-z^2| + i \operatorname{Arg}(1-z^2) = \operatorname{ln}(1-z^2),
\end{equation*}de donde:
\begin{equation*}
e^{\frac{1}{2} \operatorname{Log}(1-z^2)} = \sqrt{1-x^2} \in \mathbb{R}^+.
\end{equation*}

Así, al considerar las ramas principales de ambas funciones tenemos que:
\begin{align*}
\operatorname{sen}^{-1}(z) & = -i \operatorname{Log}\left(iz +\sqrt{1-z^2}\right)\\
& = -i \operatorname{Log}\left(iz + e^{\frac{1}{2} \operatorname{Log}(1-z^2)}\right)\\
& = -i \operatorname{Log}\left(ix + \sqrt{1-x^2}\right).
\end{align*}

Dado que $iz$ es un número imaginario puro, entonces los valores que toma $iz + e^{\frac{1}{2} \operatorname{Log}(1-z^2)}$ están en la mitad derecha del plano complejo, por lo que no están en el corte de rama de la función logaritmo. De hecho, dichos valores están sobre la mitad de la circunferencia unitaria que está en la mitad derecha del plano complejo, figura 83, ya que:
\begin{equation*}
\left| iz +\sqrt{1-z^2} \right| = \sqrt{x^2+(1-x^2)} = 1.
\end{equation*}

Por lo tanto:
\begin{align*}
\operatorname{sen}^{-1}(z) & = -i \operatorname{Log}\left(ix + \sqrt{1-x^2}\right)\\
& = -i \operatorname{ln}\left|ix + \sqrt{1-x^2}\right| + \operatorname{Arg}\left(ix + \sqrt{1-x^2}\right)\\
& = \operatorname{Arg}\left(ix + \sqrt{1-x^2}\right),
\end{align*}

donde $-\dfrac{\pi}{2}<\operatorname{Arg}\left(ix + \sqrt{1-x^2}\right)<\dfrac{\pi}{2}$, entonces:
\begin{equation*}
-\dfrac{\pi}{2}<\operatorname{sen}^{-1}(z)<\dfrac{\pi}{2},
\end{equation*}para los $z=x\in\mathbb{R}$ tales que $|x|<1$.

Figura 83: Puntos $ix + \sqrt{1-x^2}$ para $x\in\mathbb{R}$ tales que $|x|<1$.

Ejemplo 23.2.
Resolvamos la ecuación $\operatorname{tan}(z)=2$.

Solución. De acuerdo con la proposición 23.1(3), si elegimos la rama principal del logaritmo tenemos que:
\begin{align*}
\operatorname{tan}^{-1}(2) & = \dfrac{i}{2} \operatorname{log}\left(\dfrac{i+2}{i-2}\right)\\
& = \dfrac{i}{2} \operatorname{log}\left(\dfrac{-3-4i}{5}\right)\\
& = \dfrac{i}{2}\left[ \operatorname{ln}\left|\dfrac{-3-4i}{5}\right| + i \left( \operatorname{Arg}\left(\dfrac{-3-4i}{5}\right) + 2 k\pi \right)\right]\\
& = \dfrac{i}{2}\left[ \operatorname{ln}\left(1\right) + i \left( \operatorname{arctan}\left(\dfrac{4}{3}\right) -\pi + 2 k\pi \right)\right]\\
& = \dfrac{1}{2}\left[ (2 k – 1)\pi – \operatorname{arctan}\left(\dfrac{4}{3}\right) \right], \quad k\in\mathbb{Z}.
\end{align*}

Observación 23.3.
Dado que las funciones inversas de las funciones trigonométricas complejas son multivaluadas, entonces debemos ser cuidadosos al derivar estas expresiones. En general, una vez establecidas las ramas de las funciones multivaluadas raíz cuadrada y logaritmo, se puede derivar las funciones inversas dentro de su dominio de analicidad mediante la regla de la cadena.

Proposición 23.2. (Derivadas funciones trigonométricas inversas.)

  1. $\dfrac{d}{dz}\operatorname{sen}^{-1}(z) = \dfrac{1}{\sqrt{1-z^2}}$, para $z\neq \pm 1$.
  2. $\dfrac{d}{dz}\operatorname{cos}^{-1}(z) = -\dfrac{1}{\sqrt{1-z^2}}$, para $z\neq \pm 1$.
  3. $\dfrac{d}{dz}\operatorname{tan}^{-1}(z) = \dfrac{1}{1+z^2}$, para $z\neq \pm i$.
  4. $\dfrac{d}{dz}\operatorname{cot}^{-1}(z) = -\dfrac{1}{1+z^2}$, para $z\neq \pm i$.
  5. $\dfrac{d}{dz}\operatorname{sec}^{-1}(z) = \dfrac{1}{z^2\sqrt{1-\frac{1}{z^2}}}$, para $z\neq \pm 1$ y $z\neq 0$.
  6. $\dfrac{d}{dz}\operatorname{csc}^{-1}(z) = – \dfrac{1}{z^2\sqrt{1-\frac{1}{z^2}}}$, para $z\neq \pm 1$ y $z\neq 0$.

Demostración.

  1. Una vez elegida una rama de la función multivaluada $\sqrt{1-z^2}$ y una adecuada rama para la función logaritmo, por la regla de la cadena tenemos que:
    \begin{align*}
    \dfrac{d}{dz}\operatorname{sen}^{-1}(z) & = \dfrac{d}{dz} = -i \operatorname{log}\left(iz +\sqrt{1-z^2}\right)\\
    & = -i \left(\dfrac{i-\dfrac{z}{\sqrt{1-z^2}}}{iz +\sqrt{1-z^2}}\right)\\
    & = -i \left(\dfrac{i\sqrt{1-z^2} – z}{\left[iz +\sqrt{1-z^2}\right] \sqrt{1-z^2}}\right)\\
    & = \dfrac{1}{\sqrt{1-z^2}}.
    \end{align*}Donde la igualdad se mantiene siempre que se utilice la misma rama de $\sqrt{1-z^2}$ tanto en la definición de la función $\operatorname{sen}^{-1}(z)$ como en la expresión de su derivada.
  2. Se deja como ejercicio al lector.
  3. Se deja como ejercicio al lector.
  4. Se deja como ejercicio al lector.
  5. Se deja como ejercicio al lector.
  6. Se deja como ejercicio al lector.

$\blacksquare$

Ejemplo 23.3.
Consideremos a las ramas principales de la funciones multivaluadas raíz cuadrada y logaritmo y determinemos el valor de la derivada de la función $\operatorname{sen}^{-1}(z)$ en el punto $z=i$.

Solución. De acuerdo con el ejemplo 23.1, sabemos que los cortes de rama de las ramas principales de las funciones multivaluadas $\operatorname{log}(z)$ y $\sqrt{1-z^2}$ son, respectivamente:
\begin{equation*}
(-\infty,0]=\left\{z=x+iy\in\mathbb{C} : x \leq 0, y=0\right\} \quad \text{y} \quad A:= \left\{z=x+iy\in\mathbb{C} : |x|\geq 1, y=0\right\}.
\end{equation*}

Figura 84: Cortes de rama de las ramas principales de las funciones multivaluadas $\operatorname{log}(z)$ y $\sqrt{1-z^2}$.

De acuerdo con lo anterior, es claro que el punto $z=i$ no pertenece al corte de rama, de la rama principal de $\sqrt{1-z^2}$. Por otra parte, tenemos que $1-i^2 = 2$, entonces:
\begin{equation*}
i(i) + \sqrt{1-i^2} = -1+\sqrt{2},
\end{equation*}el cual no es un punto sobre el corte de rama, de la rama principal del logaritmo. Por lo tanto, de la proposición 23.2(1) se sigue que:
\begin{align*}
\left. \frac{d}{dz} \operatorname{sen}^{-1}(z) \right|_{z=i} & = \left. \dfrac{1}{\sqrt{1-z^2}}\right|_{z=i}\\
& = \dfrac{1}{\sqrt{1-i^2}}\\
& = \dfrac{1}{\sqrt{2}}.
\end{align*}

Por último, si utilizamos la rama principal de la función $\sqrt{1-z^2}$ tenemos:
\begin{align*}
\left. \sqrt{1-z^2} \right|_{z=i} = \left. e^{\frac{1}{2} \operatorname{Log}(1-z^2)} \right|_{z=i} & = e^{\frac{1}{2} \operatorname{Log}(1-i^2)} \\
& = e^{\frac{1}{2} \operatorname{Log}(2)}\\
& = e^{\frac{1}{2}\left[\operatorname{ln}|2| + i \operatorname{Arg}(2)\right]}\\
& = e^{\operatorname{ln}\left(\sqrt{2}\right)}\\
& = \sqrt{2}.
\end{align*}

Por lo que la derivada es $\dfrac{1}{\sqrt{2}}$, es decir, el resultado coincide con el valor obtenido al utilizar la fórmula de la derivada.

Ejemplo 23.4.
Consideremos ahora el punto $z=\sqrt{5}$. De acuerdo con el ejemplo anterior, es claro que dicho punto está en el corte de rama, de la rama principal de la función multivaluada $\sqrt{1-z^2}$, por lo que utilizando dicha rama no podemos obtener el valor de la función $\operatorname{sen}^{-1}(z)$ ni de su derivada en $z=\sqrt{5}$. Entonces procedemos a elegir una nueva rama para la raíz cuadrada de modo que sea posible determinar dichos valores.

Solución. Por simplicidad elegimos a la rama natural de la raíz, es decir:
\begin{equation*}
\sqrt{1-z^2} = e^{\frac{1}{2} \operatorname{Log}_{[0,2\pi)}(1-z^2)}, \quad 0 \leq \operatorname{Arg}(1-z^2) < 2\pi.
\end{equation*}

El corte de rama de esta función está dado por los $z=x+iy\in\mathbb{C}$ tales que:
\begin{align*}
0 \leq \operatorname{Arg}(1-z^2) < 2\pi & \quad \Longleftrightarrow \quad \left\{ \begin{array}{l}
\operatorname{Re}(1-z^2) = 1+y^2-x^2 \geq 0, \\
\\ \operatorname{Im}(1-z^2) = -2xy = 0.
\end{array}
\right.\\
& \\
& \quad \Longleftrightarrow \quad B:= \left\{z=x+iy\in\mathbb{C} : |x|\leq 1, y=0\right\} \cup \left\{z=x+iy\in\mathbb{C} : x=0, y\in\mathbb{R}\right\}.
\end{align*}

Figura 85: Corte de rama, de la rama natural, de la función multivaluada $\sqrt{1-z^2}$.

Para esta rama es claro que el punto $z=\sqrt{5}$ no está en su corte de rama, por tanto podemos utilizarla. Tenemos que $1-(\sqrt{5})^2 = -4$, entonces:
\begin{align*}
\operatorname{Log}_{[0,2\pi)}(-4) &= \operatorname{ln}|-4| + i\operatorname{Arg}_{[0,2\pi)}(-4)\\
&= \operatorname{ln}(4) + i\pi,
\end{align*}por lo que:
\begin{align*}
\left.\sqrt{1-z^2}\right|_{z=\sqrt{5}} & = \sqrt{1-(\sqrt{5})^2}\\
& = e^{\frac{1}{2} \operatorname{Log}_{[0,2\pi)}(-4)}\\
& = e^{\operatorname{ln}(2)} e^{i\frac{\pi}{2}}\\
&= 2i.
\end{align*}

Dado que $i \sqrt{5}+2i = i(2+\sqrt{5})$ no es un punto sobre el corte de rama, de la rama principal del logaritmo, entonces utilizaremos de nuevo dicha rama. Por lo que:
\begin{align*}
\left.\operatorname{sen}^{-1}(z)\right|_{z=\sqrt{5}} & = \left. – i\operatorname{Log}\left(iz+\sqrt{1-z^2}\right)\right|_{z=\sqrt{5}}\\
& = -i\operatorname{Log}\left(i\left(2+\sqrt{5} \right)\right)\\
& = -i\left[\operatorname{ln}\left|i\left(2+\sqrt{5} \right)\right| + i \operatorname{Arg}\left(i\left(2+\sqrt{5} \right)\right)\right]\\
& = -i\left[\operatorname{ln}\left(2+\sqrt{5}\right) + i \frac{\pi}{2}\right]\\
& = \frac{\pi}{2} -i\operatorname{ln}\left(2+\sqrt{5}\right).
\end{align*}

Mientras que:
\begin{align*}
\left. \frac{d}{dz} \operatorname{sen}^{-1}(z) \right|_{z=\sqrt{5}} & = \left. \dfrac{1}{\sqrt{1-z^2}}\right|_{z=\sqrt{5}}\\
&= \dfrac{1}{\sqrt{-4}}\\
&= \dfrac{1}{2i}\\
&= -\dfrac{i}{2}.
\end{align*}

Procediendo de forma completamente análoga que con la proposición 23.1, es posible establecer las expresiones para las funciones inversas de las funciones hiperbólicas, que al estar definidas en términos de la exponencial compleja, también resultan ser funciones multivaluadas.

Proposición 23.3. (Funciones hiperbólicas inversas.)

  1. $\operatorname{senh}^{-1}(z) = \operatorname{log}\left(z + \sqrt{z^2+1}\right)$.
  2. $\operatorname{cosh}^{-1}(z) = \operatorname{log}\left(z + \sqrt{z^2-1}\right)$.
  3. $\operatorname{tanh}^{-1}(z) = \dfrac{1}{2} \operatorname{log}\left(\dfrac{1+z}{1-z}\right)$, para $z\neq \pm 1$.
  4. $\operatorname{coth}^{-1}(z) = \dfrac{1}{2} \operatorname{log}\left(\dfrac{z+1}{z-1}\right)$, para $z\neq \pm 1$.
  5. $\operatorname{sech}^{-1}(z) = \operatorname{log}\left(\dfrac{1 + z\sqrt{\dfrac{1}{z^2}-1}}{z}\right)$, para $z\neq 0$.
  6. $\operatorname{csch}^{-1}(z) = \operatorname{log}\left(\dfrac{1 + z\sqrt{\dfrac{1}{z^2}+1}}{z}\right)$, para $z\neq 0$.

Demostración.

  1. Se deja como ejercicio al lector.
  2. Sea $z = \operatorname{cosh}(w)$. De acuerdo con la definición 22.3 tenemos que:
    \begin{equation*}
    z = \operatorname{cosh}(w) = \frac{e^{w} + e^{-w}}{2},
    \end{equation*}de donde:
    \begin{equation*}
    (e^{w})^2-2z(e^{w}) + 1 = 0.
    \end{equation*}Resolvemos la ecuación cuadrática para $e^w$ utilizando la fórmula general, entonces:
    \begin{equation*}
    e^{w} = \frac{2z + \sqrt{4(z^2-1)}}{2} = z + \sqrt{z^2-1},
    \end{equation*}donde la función multivaluada $\sqrt{z^2-1}$ determina dos raíces complejas de $z^2-1$. Por lo que, una vez establecida la rama de dicha función multivaluada, podemos utilizar la proposición 20.1(6), para $k, n\in\mathbb{Z}$, como sigue:
    \begin{align*}
    w &= \operatorname{log}\left(z + \sqrt{z^2-1}\right) + i2k\pi\\
    &= \operatorname{ln}\left|z + \sqrt{z^2-1}\right| + i\left(\operatorname{Arg}\left(z + \sqrt{z^2-1}\right) + 2m\pi\right)\\
    &= \operatorname{ln}\left|z + \sqrt{z^2-1}\right| + i\operatorname{arg}\left(z + \sqrt{z^2-1}\right)\\
    &= \operatorname{log}\left(z + \sqrt{z^2-1}\right),
    \end{align*}donde $m=k+n\in\mathbb{Z}$.
  3. Se deja como ejercicio al lector.
  4. Se deja como ejercicio al lector.
  5. Se deja como ejercicio al lector.
  6. Se deja como ejercicio al lector.

$\blacksquare$

Ejemplo 23.5.
Veamos que $\operatorname{tanh}^{-1}\left(\dfrac{1}{z}\right) = \operatorname{coth}^{-1}\left(z\right)$.

Solución. Sea $z\in\mathbb{C}\setminus\{-1,1,0\}$, entonces:
\begin{align*}
\operatorname{tanh}^{-1}\left(\dfrac{1}{z}\right) & = \dfrac{1}{2} \operatorname{log}\left(\dfrac{1+\dfrac{1}{z}}{1-\dfrac{1}{z}}\right)\\
& = \dfrac{1}{2} \operatorname{log}\left(\dfrac{z+1}{z-1}\right)\\
& = \operatorname{coth}^{-1}\left(z\right).
\end{align*}

Proposición 23.4. (Derivadas funciones hiperbólicas inversas.)

  1. $\dfrac{d}{dz}\operatorname{senh}^{-1}(z) = \dfrac{1}{\sqrt{1+z^2}}$, para $z\neq \pm i$.
  2. $\dfrac{d}{dz}\operatorname{cosh}^{-1}(z) = \dfrac{1}{\sqrt{z^2 – 1}}$, para $z\neq \pm 1$.
  3. $\dfrac{d}{dz}\operatorname{tanh}^{-1}(z) = \dfrac{1}{1-z^2}$, para $z\neq \pm 1$.
  4. $\dfrac{d}{dz}\operatorname{coth}^{-1}(z) = \dfrac{1}{1-z^2}$, para $z\neq \pm 1$.
  5. $\dfrac{d}{dz}\operatorname{sech}^{-1}(z) = -\dfrac{1}{z^2\sqrt{\dfrac{1}{z^2}-1}}$, para $z\neq \pm 1$ y $z\neq 0$.
  6. $\dfrac{d}{dz} \operatorname{csch}^{-1}(z) = – \dfrac{1}{z^2\sqrt{\dfrac{1}{z^2}+1}}$, para $z\neq \pm i$ y $z\neq 0$.

Demostración.

  1. Se deja como ejercicio al lector.
  2. Una vez establecida una rama de la función multivaluada $\sqrt{z^2-1}$ y una adecuada rama para la función logaritmo, procedemos a derivar utilizando la regla de la cadena, entonces:
    \begin{align*}
    \dfrac{d}{dz}\operatorname{cosh}^{-1}(z) & = \dfrac{d}{dz} \operatorname{log}\left(z + \sqrt{z^2-1}\right)\\
    & = \dfrac{1+\dfrac{z}{\sqrt{z^2-1}}}{z + \sqrt{z^2-1}}\\
    & = \dfrac{z +\sqrt{z^2-1}}{\left(z +\sqrt{z^2-1} \,\right) \sqrt{z^2-1}}\\
    & = \dfrac{1}{\sqrt{z^2-1}}.
    \end{align*}Donde la igualdad se mantiene siempre que se utilice la misma rama de $\sqrt{z^2-1}$ tanto en la definición de la función $\operatorname{cosh}^{-1}(z)$ como en la expresión de su derivada.
  3. Se deja como ejercicio al lector.
  4. Se deja como ejercicio al lector.
  5. Se deja como ejercicio al lector.
  6. Se deja como ejercicio al lector.

$\blacksquare$

Ejemplo 23.6.
Utilizando la rama natural de la función $\sqrt{z^2-1}$ y la rama principal del logaritmo determinemos:
a) $\operatorname{cosh}^{-1}\left(\dfrac{1}{\sqrt{2}}\right)$.
b) $\dfrac{d}{dz}\operatorname{cosh}^{-1}\left(\dfrac{1}{\sqrt{2}}\right)$.

¿Es posible determinar los valores de cada inciso si se considera la rama principal de la función $\sqrt{z^2-1}$?

Solución. De acuerdo con el ejercicio 13.15, sabemos que para la función multivaluada $\sqrt{z^2-1}$ los cortes de rama, considerando las ramas principal y natural son, respectivamente:
\begin{equation*}
\left\{z=x+iy\in\mathbb{C} : |x|\leq 1, y=0\right\} \quad \text{y} \quad \left\{z=x+iy\in\mathbb{C} : |x|\geq 1, y=0\right\}.
\end{equation*}

Dado que $z=\dfrac{1}{\sqrt{2}}$ y $|z|<1$, entonces dicho punto está sobre el corte de rama, de la rama principal de la función $\sqrt{z^2-1}$, por tanto no podemos utilizar dicha rama para determinar los valores que nos pide cada inciso.

Por otra parte, es claro que $z=\dfrac{1}{\sqrt{2}}$ no está sobre el corte de rama, de la rama natural de la función $\sqrt{z^2-1}$, entonces:
\begin{equation*}
\sqrt{1-z^2} = e^{\frac{1}{2} \operatorname{Log}_{[0,2\pi)}(z^2-1)}, \quad 0 \leq \operatorname{Arg}(z^2-1) < 2\pi.
\end{equation*}

Tenemos que:
\begin{align*}
\operatorname{Log}_{[0,2\pi)}\left(\left(\dfrac{1}{\sqrt{2}}\right)^2 – 1\right) = \operatorname{Log}_{[0,2\pi)}\left(-\dfrac{1}{2}\right)
&=\operatorname{ln}\left|-\dfrac{1}{2}\right| + i\operatorname{Arg}_{[0,2\pi)}\left(-\dfrac{1}{2}\right)\\
&= -\operatorname{ln}(2) + i \pi,
\end{align*}por lo que:
\begin{align*}
\left.\sqrt{z^2-1}\right|_{z=\frac{1}{\sqrt{2}}} & = \sqrt{\left(\dfrac{1}{\sqrt{2}}\right)^2 – 1}\\
& = e^{\frac{1}{2} \operatorname{Log}_{[0,2\pi)}\left(-\frac{1}{2}\right)}\\
& = e^{-\frac{\operatorname{ln}(2)}{2}} e^{-\frac{\pi}{2}}\\
&= i\frac{1}{\sqrt{2}}.
\end{align*}

Dado que $\dfrac{1}{\sqrt{2}} + i\dfrac{1}{\sqrt{2}} = \dfrac{1}{\sqrt{2}}\left(1+i\right)$ no es un punto sobre el corte de rama, de la rama principal del logaritmo, entonces utilizaremos de nuevo dicha rama. Por lo que:
\begin{align*}
\left.\operatorname{cosh}^{-1}(z)\right|_{z=\frac{1}{\sqrt{2}}} & = \left. \operatorname{Log}\left(z+\sqrt{z^2-1}\right)\right|_{z=\frac{1}{\sqrt{2}}}\\
& =\operatorname{Log}\left( \dfrac{1}{\sqrt{2}}\left(1+i\right)\right)\\
& =\operatorname{ln}\left| \dfrac{1}{\sqrt{2}}\left(1+i\right)\right| + i \operatorname{Arg}\left( \dfrac{1}{\sqrt{2}}\left(1+i\right)\right)\\
& = \operatorname{ln}\left(1\right) + i \frac{\pi}{4}\\
& = i\frac{\pi}{4}.
\end{align*}

Mientras que:
\begin{align*}
\left. \frac{d}{dz} \operatorname{cosh}^{-1}(z) \right|_{z=\frac{1}{\sqrt{2}}} & = \left. \dfrac{1}{\sqrt{z^2-1}}\right|_{z=\frac{1}{\sqrt{2}}}\\
&= \dfrac{1}{\sqrt{-\frac{1}{2}}}\\
&= \dfrac{1}{\frac{i}{\sqrt{2}}}\\
&= -i\sqrt{2}.
\end{align*}

Tarea moral

  1. Completa las demostraciones de las proposiciones de esta entrada.
  2. Sean $w=\operatorname{cos}(z)$ y $\zeta = e^{iz}$. Muestra que:
    \begin{align*}
    \zeta & = w+\sqrt{w^2-1},\\
    \operatorname{cos}^{-1}(w) & = -i\operatorname{log}\left(w\pm \sqrt{w^2-1}\right).
    \end{align*}
  3. Muestra que los puntos de ramificación de la función multivaluada $\operatorname{sen}^{-1}(z)$ son $z=\pm 1$.
  4. Demuestra que si $a\in\mathbb{R}$ y $a>1$, entonces:
    \begin{equation*}
    \operatorname{tanh}^{-1}(a) = \operatorname{Log}\sqrt{\frac{a+1}{a-1}} + i \frac{2k+1}{2} \pi, \quad k\in\mathbb{Z}.
    \end{equation*}
  5. Muestra que si se usa la misma rama de la función $\sqrt{1-z^2}$ en la definición de las funciones multivaluadas $\operatorname{sen}^{-1}(z)$ y $\operatorname{cos}^{-1}(z)$, proposición 23.1, entonces:
    a) $\operatorname{sen}^{-1}(z) + \operatorname{cos}^{-1}(z) = 2k\pi +\frac{\pi}{2}, \,\, k\in\mathbb{Z}$.
    b) $\operatorname{tan}^{-1}(z) + \operatorname{cot}^{-1}(z) = k\pi -\frac{\pi}{2}, \,\, k\in\mathbb{Z}$.
  6. Resuelve las siguientes ecuaciones:
    a) $\operatorname{senh}(5z+i) = -\sqrt{3} i$.
    b) $\operatorname{tanh}\left(\frac{z-3}{2}\right) = -1+ i$.
    c) $\operatorname{cot}(z) = 2i$.
    d) $\operatorname{cosh}^2(z) = -1$.
  7. Prueba que:
    \begin{equation*}
    \operatorname{tanh}^{-1}\left(e^{i\theta}\right) = \frac{1}{2} \operatorname{log}\left(i \operatorname{cot}\left(\frac{\theta}{2}\right)\right).
    \end{equation*}Determina una expresión similar para $\operatorname{tan}^{-1}\left(e^{i\theta}\right)$.
  8. Demuestra que:
    \begin{equation*}
    \operatorname{tan}\left(i \operatorname{log}\left(\frac{a-ib}{a+ib}\right)\right) = \frac{2ab}{a^2 – b^2}.
    \end{equation*}Hint: sustituye $z$ por $\dfrac{2ab}{a^2 – b^2}$ en la definición de $\operatorname{tan}^{-1}(z)$.
  9. Determina los puntos de ramificación de las siguientes funciones:
    a) $\operatorname{cos}^{-1}(z)$.
    b) $\operatorname{tan}^{-1}(z^2+2z+1)$.

Más adelante…

En esta entrada hemos abordado de manera general las definiciones de las funciones inversas de las funciones trigonométricas e hiperbólicas. Vimos que estas funciones resultan ser funciones multivaluadas, por lo que es importante recordar los conceptos de la entrada 13 referentes a este tipo de funciones, como los conceptos de rama de una función multivaluada, corte de rama y puntos de ramificación, ya que a través de estos conceptos es posible determinar de manera clara los dominios de analicidad de dichas funciones. Así mismo, vimos que una vez definida una rama de alguna de estas funciones inversas, es posible determinar su derivada a través de la regla de la cadena.

La siguiente entrada abordaremos el concepto de transformación, que como hemos visto en nuestros cursos de Geometría y Álgebra Lineal resulta ser una herramienta muy útil para el estudio de funciones de varias variables, en este caso para las funciones complejas, ya que a través de dicho concepto podremos dar una interpretación geométrica del comportamiento de las funciones complejas.

Entradas relacionadas